SlideShare ist ein Scribd-Unternehmen logo
1 von 29
Disorder of hair and nails
1) Fungal finger onycholysis usually results from:

A. Fusarium spp

B. Trichophyton rubrum

C. Candida albicansCorrect Choice

D. Scopulariopsis brevicaulis

E. Trichophyton interdigitale
It is widely accepted that C. albicans acts as colonizer that has found an ideal environment.


2) The visible portion of the nail matrix is called the:

A. Hyponychium

B. Eponychium

C. LunulaCorrect Choice

D. Nail plate

E. Cuticle
The nail unit is comprised of six major components including the nail matrix, nail platic, cuticle, nail
bed, anchoring portion, and the framing portion (proximal/lateral/distal folds). The lunula is a
component of the distal matrix. It is grayish white because of the nature of the keratinization of its
epithelium


3) Steatocystoma multiplex and natal teeth are associated with which of the following conditions?

A. Rubenstein-Taybi syndrome

B. Nail-patella syndrome

C. Jackson-Sertole pachyonychia congenita (Type 2) Correct Choice

D. Jadassohn-Lewandowsky pachyonychia congenita (Type 1)

E. Coffin-Siris syndrome
Jackson-Sertole pachyonychia congenita (Type 2) is characterized by thickening of the nailbed and
plate, steatocystoma multiplex, and natal teeth


4) Atrophy of the isthmus is a characteristic histologic finding of which of the following conditions?

A. Folliculitis Decalvans

B. Parry-Romberg syndrome

C. Discoid lupus erythematosus

D. Lichen planopilaris Correct Choice

E. Acne Keloidalis Nuchae
Lichen planopilaris is characterized by perifollicular lymphocytic infiltration with atrophy of the
isthmus
5) A patient with diffuse severe sudden hair loss developing 3 months after hospitalization for
septicemia likely has which of the following conditions?


A. Uncombable hair syndrome

B. Catagen effluvium

C. Loose anagen syndrome

D. Anagen effluvium

E. Telogen effluvium Correct Choice
Telogen effluvium typically ensues stressful events after a 2-3 month period


6) Primary total dystrophic onychomycosis is due to:

A. Trichophyton rubrum

B. Candida sppCorrect Choice

C. Fusarium

D. Scytalidium dimidiatum

E. Scopulariopsis brevicaulis
The main characteristic of this immunologic disorder the simultaneous involvement of all the tissues
of the nail unit


7) An 18 year-old girl who was hospitalized last month after a serious car accident is noted to have
white transverse grooves on her fingernails and toenails. The most likely diagnosis is:


A. Half-and-half nails

B. Twenty-nail dystrophy

C. Terry’s nails

D. Mees’ lines

E. Beau’s lines Correct Choice
Beau’s lines are transverse grooves in the nails. They are caused by a generalized systemic
condition which disrupts nail formation. Conditions may include infection, myocardial infection,
neurologic events and cytotoxic medications.


8) Papular atrichia is caused by mutations in which gene?

A. Patched

B. Wingless

C. Fox forehead

D. Hairless Correct Choice

E. Distal-less homebobox
Hairless gene mutations cause papular atrichia.


9) This nail finding is characteristically associated with what disorder?
A. Congenital heart failure

B. Hyperthyroidism

C. Cirrhosis

D. Chronic renal failureCorrect Choice

E. Anemia
Lindsay’s nail is characteristically associated with chronic renal failure


10) Koilonychia is most commonly seen in association with:

A. Progressive systemic sclerosis

B. Renal disease

C. Impaired iron metabolismCorrect Choice

D. Pulmonary disease

E. Alopecia areata
In koilonychia (spoon nails), the nail is a concave shape with raised lateral
edges. Iron-deficiency anemia and Plummer-Vinson syndrome have been associated with
koilonychia. Involvement of only the first three fingernails is suggestive of an occupational cause. In
early infancy, koilonychia is a normal finding. Koilonychia is also seen in hemochromatosis, an
autosomal recessive condition associated with increased intestinal iron absorption and increased
iron deposition leading to metallic-grey hyperpigmentation of the skin, hepatomegaly, diabetes,
hypogonadism, polyarthritis, and cardiac abnormalities


11) What condition most likely describes a hair defect with ruffled cuticle in 4-year old female with
blond hair?


A. Loose anagen hair syndromeCorrect Choice

B. Pili annulati

C. Pili torti

D. Wooly hair

E. Monilethrix
Loose anagen hair syndrom occurs primarily in young, females with blond hair. The defect results in
early keratinization of the inner root sheath which leads to poor anchoring. Examination of hair
shaft reveals ruffled proximal cuticle


12) How long should last the wash-out period before performing mycologic investigations in a
clinically recurrent onychomycosis treated by terbinafine, itraconazole or fluconazole:


A. Four months

B. Two months

C. Three monthsCorrect Choice

D. Six months

E. One month
This is an interesting point, because we know that these systemic antifungals are retained in the
keratin of the toenails up to six months after the treatment has been stopped.
Therefore this answer could have been debatable were it not for the presence of fungi in the nail
bed (which is the primary location of the disease) and where the scraping should be properly
performed for mycologic testing after removal of a piece of nail as proximal as possible


13) Which of the following is NOT a cause of red lunulae?

A. EndocarditisCorrect Choice

B. Carbon monoxide poisoning

C. Rheumatoid arthritis

D. Imuran

E. Psoriasis
Endocarditis is not a cause of red lunulae. Alopecia areata, psoriasis, prednisone for connective
tissue diseases, carbon monoxide poisoning, cardiac failure, SLE, RA, COPD, CO2, cirrhosis, imuran
and radiation are the causes of this condition


14) The "ragged cuticle" seen in dermatomyositis is also known as:

A. Emperipolesis

B. Plummers sign

C. ventral pterygium

D. Candy-cane nails

E. Samitz signCorrect Choice
Cuticular fraying from dermatomyositis is called the Samitz sign or "ragged cuticle". The other
options are not associated with dermatomyositis. A ventral pterygium can be seen in systemic
sclerosis. There is no such sign as Plummers sign. Candy-cane nails are seen in Darier's disease.
Emperipolesis is seen in Rosai-Dorfman disease and is not a nail finding


15) Acute paronychia is most commonly caused by which of the following organisms?

A. Trichophyton rubrum

B. Staphylococcus aureus Correct Choice

C. Candida tropicalis

D. Candida albicans

E. Pseudomonas aeruginosa
Staphylococcus aureus is the most common cause of acute paronychia


16) The combination of poliosis, uveitis, deafness, and vitiligo are characteristic of which of the
following conditions?


A. Vogt-Koyanagi-Harada syndrome Correct Choice

B. Cornelia de Lange Syndrome

C. Piebaldism

D. Waardernburg syndrome

E. Cronkhite-Canada syndrome
Vogt-Koyanagi-Harada syndrome is characterized by poliosis, uveitis, deafness, and Vitiligo
17) A patient with curly hair that straightened after puberty, enamel hypoplasia, dental pits, and
increased bone density likely has a mutation in which of the following genes?


A. SPINK 5

B. Distal-less homeobox-3 gene (DLX3) Correct Choice

C. SLURP 1

D. Bone morphogenetic protein type 2

E. Vascular-endothelial growth factor receptor 3
Distal-less homeobox-3 gene (DLX3) mutations cause trichodentoosseous syndrome, which is
characterized by curly hair that straightens in the 2nd-3rd decades, enamel hypoplasia, dental pits,
and increased bone density


18) Argininosuccinic aciduria is characteristically associated with this hair finding –name the
condition pictured below:


A. Trichoschisis

B. Monilethrix

C. Pili trianguli et canaliculi

D. Trichorrhexis nodosa Correct Choice

E. Trichorrhexis invaginata
Trichorrhexis nodosa is a finding in argininosuccinic aciduria


19) Signs of virilization include all of the following EXCEPT:

A. Increased muscle mass in the limb girdles

B. Clitoromegaly

C. Temporal balding

D. Centripetal obesityCorrect Choice

E. Acne
Centripetal obesity is a sign of cortisol excess not of virilization. In addition to the those listed
above, irregular or absent menses and deepening of the voice are signs of virilization


20) Distal subungual onychomycosis is most often caused by:

A. Epidermophyton floccosum

B. Trichophyton megninii

C. Trichophyton mentagrophytes

D. Trichophyton rubrumCorrect Choice

E. Trichophyton schoenleinii
Trichophyton rubrum is the most common organism involved in distal subungual onychomycosis.


21) Yellow nails can be seen in each of the following syndromes except:
A. Bronchiectasis

B. HyperlipidemiaCorrect Choice

C. Thyroid disease

D. Penicillamine

E. Tobacco use
Yellow discoloration of the nail plate can be associated with bronchiectasis or underlying pulmonary
disease such as COPD. Other associated conditions include lymphedema, rheumatoid arthritis,
thyroid disease, malignancies, penicillamine, and chronic sinusitis or bronchitis. Hyperlipidemia is
not associated with nail changes


22) Lindsay's nails (distal nail normal, proximal nail white) is characteristic of:

A. Plummer-Vinson syndrome

B. LEOPARD syndrome

C. Hemochromatosis

D. Chronic renal failureCorrect Choice

E. Ectodermal dysplasia
Lindsay's nails are characteristic of chronic renal failure and indicate nailbed edema. The remaining
options are causes of koilonychia, a spooning defect of the nail


23) A woman with hypopigmented in lines of Blaschko and scarring alopecia likely suffers which of
the following conditions?


A. Chondrodysplasis punctata

B. Anhidrotic Ectodermal Dysplasia

C. Bloch-Sulzberger Syndrome Correct Choice

D. Rothmund-Thomson Syndrome

E. Focal Dermal Hypoplasia
Bloch-Sulzberger Syndrome (incontinentia pigmenti)


24) The most common cause for superficial type onychomycosis is:

A. Epidermophyton floccosum

B. Scopulariopsis brevicalis

C. Trichophyton rubrum

D. Fusarium oxysporum

E. Trichophyton mentagrophytesCorrect Choice
T. mentagrophytes is the most common cause of superficial type onychomycosis. Other causes
include: Fusarium, Acremonium and Aspergillus spp


25) Congenital generalized hypertrichosis is associated with which of the following symptoms?

A. Methimazole teratogenicity
B. Gingival fibromatosis Correct Choice

C. Androgen-secreting ovarian tumors

D. Pituitary insufficiency

E. Distichiasis
Gingival fibromatosis. Congenital Generalized Hypertrichosis with Gingival Fibromatosis is a genetic
condition with hypertrichosis on the face and upper body and gingival hyperplasia and fibromatosis


26) A significantly increased number of "club hairs" on a pull test is indicative of:

A. Anagen effluvium

B. Alopecia areata

C. Angrogenetic alopecia

D. Telogen effluviumCorrect Choice

E. Normal hair anatomy
Telogen hairs are characteristically club shaped. An increased number of telogen hairs on hair pull
would be suggestive of telogen effluvium. Anagen hairs have a curled appearance at the root. While
a small number of telogen hairs are normal, a significantly increased number would not be normal
hair anatomy


27) Trichobezoar is is a potentially life-threatening complication of:

A. Discoid lupus

B. Anhidrotic ectodermal dysplasia

C. Alopecia areata

D. Lichen planopilaris

E. Trichotillomania Correct Choice
NEEDS EXPLANATIONS


28) Which of the following is an example of an apparent leukonychia?

A. Mee's lines

B. Terry's lines

C. Onychoschizia

D. Hutchinson's lines

E. Muehrcke's linesCorrect Choice
An apparent leukonychia disappears with pressure and is a problem with nail bed, not nail plate.
Apparent leukonychia may be indicative of overall health compromise. Muehrcke's lines are paired
white bands and are a type of apparent leukonychia


29) Endonyx onychomycosis is due to:

A. Trichophyton soudanenseCorrect Choice

B. Trichophyton rubrum
C. Scytalidium dimidiatum

D. Scopuliaropsis

E. Trichophyton mentagrophytes
Endonyx onychomycosis may also be observed with T. violaceum. This is the equivalent of endothrix
infection of the hair associated with tinea capitis


30) A patient with yellow nail syndrome should be evaluated for systemic disease involving which
organ system?


A. Hematologic

B. Hepatic

C. Pulmonary Correct Choice

D. Cardiovascular

E. Renal
Yellow nail syndrome is characterized by slowly growing, yellow nails that are excessively curved.
The lunulae and cuticles are absent. It has been associated with a array of pulmonary diseases
including tuberculosis, asthma, pleural effusion, bronchiectasis, sinusitis, bronchitis, and COPD


31) The defect associated with the hair finding shown in the image is:

A. hHB6/hHB1Correct Choice

B. hHB8/hHB3

C. hHB10/hHB5

D. hHB9/hHB4

E. hHB7/hHB2
hHB6/hHB1 keratins are associated with Monilethrix. Other findings include: short, brittle hair,
extensor keratosis pilaris, mental retardation, syndactyly, cataracts, and nail/teeth abnormalities


32) Which of the following syndromes does not have Dorsal pterygium as a feature:

A. Lesch-Nyhan syndrome

B. Lamellar ichthyosisCorrect Choice

C. Cicatricial pemphigoid

D. Chronic GVHD

E. Lichen planus
Dorsal pterygium (scarring of the proximal nail fold) is caused by lichen planus, acrosclerosis,
onychotillomania, Lesch-Nyhan syndrome, chronic GVHD, SJS/TEN and cicatricial pemphigoid. All of
the listed items are correct except lamellar ichthyosis. It does not have this nail finding.


33) Which part of the matrix gives rise to the dorsal nail plate?

A. Distal matrix

B. Dorsal matrix

C. Ventral matrix
D. Lateral matrix

E. Proximal matrix Correct Choice
The proximal matrix gives rise to the dorsal nail plate, while the distal matrix gives rise to the
ventral nail plate


34) Yellow nail syndrome can be associated with:

A. D-penicillamine use in patients with rheumatoid arthritisCorrect Choice

B. Alopecia areata

C. Wilson’s disease

D. Doxorubicin

E. Minocycline use in acne patients
Answer: A. D-penicillamine use in patients with rheumatoid arthritis.
Yellow nail syndrome has been described in patients with rheumatoid arthritis receiving thiol
compounds including D-penicillamine and bucillamine. Reversal of the condition has been described
after discontinuation of the drug


35) A patient with cirrhosis has noticed that the proximal aspect of his nails have become white.
This characteristic finding is called:


A. Muehrcke's lines

B. Mees' lines

C. Terry's linesCorrect Choice

D. Beau's lines

E. Half-and-half nails
Terry’s lines are described as an abnormal white appearance of the nail except for the most distal
portion. It has been described in association with cirrhosis, congestive heart failure, and diabetes
mellitus


36) A defect in which part of the nail cause a true leukonychia?

A. Lunula

B. Nail plateCorrect Choice

C. Hyponychium

D. Nail bed

E. Cuticle
True leukonychia is caused by an acute stress to the nail and manifests in the nail plate. Examples
include Terry's nails and Mee's lines. Apparent leukonychia, on the other hand, are a result of
problems in the nail bed and are indicative of overall health compromise. Examples of apparent
leukonychia include Lindsay's and Muehrcke's nail


37) Tay’s syndrome is characterized by this hair condition pictured below – identify the condition:

A. Monilethrix

B. TrichoschisisCorrect Choice
C. Trichorrhexis nodosa

D. Trichorrhexis invaginata

E. Pili trianguli et canaliculi
Tay’s syndrome is also known as trichothiodystrophy which is characterized by trichoschisis or clean
fracture of the hair shaft


38) A patient with wooly hair, diffuse palmoplantar keratoderma, and right ventricular
arrhythmogenic cardiomyopathy likely has a mutation in which of the following genes?


A. HHB6

B. Plakophilin

C. SPINK 5

D. Plakoglobin Correct Choice

E. SLURP 1
Naxos syndrome is caused by mutations in the plakoglobin gene. Its clinical features include wooly
hair, diffuse palmoplantar keratoderma, and right ventricular arrhythmogenic cardiomyopathy


39) Black nail is caused by:

A. Proteus mirabilisCorrect Choice

B. Staphylococcus aureus

C. Hortaea werneckii

D. Pseudomonas spp.

E. Trichophyton mentagrophytes
Nail infections with Proteus mirabilis. T. mentag causes white discoloration of the nail and
Pseudomonas will cause a green or yellow/green discoloration. Hortaea werneckii causes tinea
nigra. S. aureus does not cause black nails


40) The combination of a low-set hairline and synophrys is seen in which of the following
conditions?


A. Noonan Syndrome

B. Turner Syndrome

C. Waardenburg's syndrome

D. Cornelia de Lange Syndrome Correct Choice

E. Werner syndrome
Explanation


41) Pili trianguli et canaliculi is the characteristic hair fiding in which of the following disoders?

A. Monilethrix

B. Naxos syndrome

C. Anhidrotic ectodermal dysplasia
D. Pili annulati

E. Uncombable hair syndrome Correct Choice
Pili trianguli et canaliculi, characterized by a triangular hair shaft with a central longitudinal groove,
is the finding seen in uncombable hair syndrome


42) The most common cultured agent in chronic paronychia associated with a black hue of the
lateral edge of the nail plate is:


A. Trichophyton rubrum

B. gram-negative bacteria

C. Scytalidium dimidiatum

D. Scopulariopsis brevicaulis

E. Candida spp.Correct Choice
Chronic paronychia is likely a condition that represents a contact reaction to allergens or irritants. It
often is superinfected with candida. This underlies the importance of treatment with both an
antifungal and a topical steroid


43) Cutaneous signs of cortisol excess include all of the following EXCEPT:

A. Atrophic/fragile skin

B. Dorsocervical/Supraclavicular fat pads

C. Striae distensae

D. Plethora

E. Peripheral obesityCorrect Choice
All of the listed options are signs of cortisol excess except peripheral obesity. The most common
pattern of obesity in patients with cortisol excess is centripetal or central obesity


44) Which genetic defect could be expected in an individual with these nail findings in addition to
short stature, capillary malformations, beaked nose, mental retardation, and cryptorchidism?


A. ectodysplasin

B. hHB1/hHB6

C. GNAS

D. CREBBPCorrect Choice

E. ERCC6
Rubenstein-Taybi syndrome is a genetic disorder caused by mutations in the gene encoding CREB
binding protein. This protein acts as a nuclear protein involved in coactivating cAMP regulated gene
expression. Clinically, patients have capillary capillary malformations, beaked nose, mental
retardation, cryptorchidism, and broad thumbs


45) Which autoantibody is associated with these cuticular changes, shawl sign, and overall
favorable prognosis?


A. Scl-70

B. High titered ANA with speckled pattern
C. Mi-2Correct Choice

D. Ro

E. Jo-1
Dermatomyositis has many different cutaneous manifestations including heliotrope rash, Gottron's
papules and sign, mechanic's hand and poikiloderma atrophicans vasculare. Antibodies to Mi-2 in
dermatomyositis are associated with cuticular changes, shawl sign and a overall favorable prognosis


46) The most common nail infection that causes white superficial/proximal white subungal
onychomycosis is:


A. Trichophyton rubrumCorrect Choice

B. Trichophyton mengatrophytes

C. Scatylidium hyalinum

D. Aspergillus spp

E. Hendersonula toruloidea
T. rubrum is the most common cause of white superficial or proximal white subungal
onychomycosis. This can also be caused by aspergillus. T. mentag causes superficial onychomycosis
and less commonly distal subungal type. Hendersonula and Scatylidium cause lateral invasion with
paronychia


47) Acquired progressive kinking of hair is a complication of which of the following medications?

A. Slfonamides

B. Antimetabolites

C. Antimalarials

D. Dapsone

E. Retinoids Correct Choice
Retinoids can cause acquired progressive kinking of hair


48) Follicular atrophoderma is a feature of which of the following conditions?

A. Papular atrichia

B. Tricho-rhino-phalangeal syndrome

C. Menkes kinky hair syndrome

D. Tricho-dento-osseous syndrome

E. Bazex syndrome Correct Choice
Bazex syndrome is characterized by follicular atrophoderma, hypotrichosis, hypohidrosis, and
mutiple BCCs of face.


49) Proximal white subungual onychomycosis with polydactylous involvement needs at least one
laboratory test:


A. HIVCorrect Choice

B. T-helper CD4 lymphocytes
C. Blood count

D. Sedimentation rate

E. Liver enzymes
This type of onychomycosis has a prognostic value: It means that the CD4 lymphocytes are below
450.


50) Yellow nail syndrome is associated with which of the following?

A. Diabetes mellitus

B. Dermatophyte infection

C. Multiple myeloma

D. Lymphedema and bronchiectasis Correct Choice

E. Panhypopituitarism
Lymphedema and bronchiectasis are associated with yellow nail syndrome


51) This hair disorder and mutations in ATP7A (MNK) gene are found in what genodermatosis?

A. Crandall's syndrome

B. Bjornstad syndrome

C. Trichothiodystrophy

D. Menke's kinky hair syndrome Correct Choice

E. Bazex's follicular atrophoderma
Menke's kinky hair syndrome is characterized by pili torti and mutations in APT7A (MNK) gene.
Bjornstad syndrome, Crandall’s syndrome and Bazex’s follicular atrophoderma are associated with
pili torti but are not associated with this mutation


52) Which of the following conditions with hair abnormalities is caused by mutations in a DNA
helicases?


A. Anhidrotic Ectodermal Dysplasia

B. KID Syndrome

C. Rothmund-Thomson Syndrome Correct Choice

D. Hidrotic Ectodermal Dysplasia

E. Incontinentia Pigmenti
Explanation


53) Pathognomonic nails changes in HOOD syndrome or Hereditary osteoonychodysplasia are

A. Pterygium inversum

B. Triangular lunulaeCorrect Choice

C. Yellow nails

D. Red and white longitudinal banding
E. Pincer nails
In the HOOD syndrome (Hereditary osteoonychodysplasia) or the Nail-Patella syndrome, patients
have different degrees of nail dysplasia. In general, the thumbs are most severely involved and nail
disease decreases in severity from the second to the fifth digits. Characteristic nail changes include
longitudinal ridging, koilonychia, and splitting. Toenails are rarely involved. Pathognomonic nail
change is the V shaped triangular lunulae with the distal peak in the midline


54) What is the mode of inheritance of this disorder of the hair shaft?

A. Sporadic

B. Autosomal recessive

C. X-linked recessive

D. Autosomal dominantCorrect Choice

E. X-linked dominant
Monilethrix is an autosomal dominant condition with variable penetrance. The hair shaft has an
elliptical or beaded hair appearance


55) Triangular lunulae are:

A. associated with a renal cell carcinoma

B. associated with hyperplastic patellae

C. part of the syndrome caused by a LMX1b mutationCorrect Choice

D. associated with a mutation of SERCA2

E. associated with a mutation of ATP2C1
Triangular lunulae are part of the Nail-Patella syndrome. Findings in this syndrome include LMX1b
mutation, hypoplastic or absent patellae, nephropathy, a "Lester Iris" - hyperpigmentation of the
pupillary margin, cataracts, heterochromia irides and glaucoma. ATP2C1 and SERCA2 are associated
with Hailey-Hailey and Darier-White disease respectively.


56) Fraying of the cuticles is a sign of which of the following conditions?

A. Lead poisoning

B. Lupus erythematosus

C. Dermatomyositis Correct Choice

D. Systemic sclerosis

E. Hypothyroidism
Dermatomyositis can be associated with fraying of the cuticles (Samitz sign).


57) Frontal bossing, saddle nose, hypoplastic midface, peg shaped or conical teeth and
hypopigmented short sparse scalp and body hair are prominent features of which of the following


A. Hidrotic ectodermal dysplasia

B. Arginosuccinic aciduria

C. Monilethrix
D. Christ Siemens Touraine syndromeCorrect Choice

E. Pachyonychia congenita
Chrsit Siemens Touraine syndrome also known as Hypohidrotic ectodermal dysplaisa or anhidrotic
ectodermal dysplasia is X-linked recessive disroder that presents in infancy to early childhood.
Features include hypo-anhidrosis with increased body temperature, hypopigmented sparse scalp
and body hair, frontal bossing, saddle nose, hypoplastic midface, abnormal ears, hypo-anodontia,
and increased bronchopulmonary inections. Cloustons syndrome or hidrotic ectodermal dysplasia is
AD and associated with palmoplantar keratoderma, nail dystrophy, short sparse scalp hair, and
tufting of the terminal phalanges. Arginosuccinic aciduria is associated with seizures, trichorrhexis
nodosa, failure to thrive, and hyperammonemia. Monelothrix is associated with beaded hairs and
keratosis pilaris. Pachyonychia congenita is associated with nail dystrophy and focal palmoplantar
keratoderma


58) Which of the following syndromes does NOT have the finding shown in the image as part of the
spectrum?


A. CrandallCorrect Choice

B. Argininosuccinic aciduria

C. Citrullinemia

D. Menke's Kinky Hair

E. Netherton
Crandall syndrome has pili torti as its hair finding. The other options all have trichorrhexis nodosa as
one of the hair findings in the syndrome. Trichorrhexis nodosa is the fraying of hair ends so that the
resemble a "broom-stick" or "paint brush"


59) Which of the following is caused by mutations in gap junction proteins?

A. Hidrotic ectodermal dysplasiaCorrect Choice

B. Anhidrotic ectodermal dysplasia

C. Netherton syndrome

D. Naxos syndrome

E. Rothmund-Thompson syndrome
Hidrotic ectodermal dysplasia is caused by mutations in connexin 30, which is a gap junction protein


60) Proximal white subungual onychomycosis associated with paronychia is never due to:

A. Fusarium spp

B. Candida spp.Correct Choice

C. Scopulariopsis brevicaulis spp

D. Acremonium spp

E. Aspergillus spp.
All of the molds indicated, here, may be responsible for this condition, but principally 2,3,4


61) Muehrcke's nails are most often associated with

A. Hyperalbuminemia
B. Hyperthyroidism

C. HypoalbuminemiaCorrect Choice

D. Hypothyroidism

E. None of these answers are correct
Muehrcke's nails are most associated with hypoalbuminemia. They represent transverse double
white lines that are the abnormality of the vascular bed, probably a localized edematous state
secondary to the hypoalbuminemia


62) Anonychia is not a feature of which of the following syndromes?

A. Hidrotic ectodermal dysplasiaCorrect Choice

B. DOOR syndrome

C. Nail-Patella syndrome

D. Coffin-Siris syndrome

E. COIF syndrome
Hidrotic ectodermal dysplasia features hyperconvex nails, micronychia and nail dystrophy. All of the
other listed options can cause anonychia. COIF syndrome is Conginital Onychodysplasia of the Index
Finger and DOOR syndrome is Deafness, Congenital Onychodystrophy Recessive Form. Coffin-Siris
syndrome is characterized by: nail onychodysplasia or aplasia (usually of the fifth finger or toe),
coarse facies including bushy eyebrows, scant scalp hair, full lips, and microcephaly, mental/growth
deficiency, short distal phalanges, and other abnormalities


63) A 4 year-old boy has a single circular alopecic atrophic patch on the vertex of the scalp. This
most likely is due to which of the following conditions?


A. Triangular Alopecia

B. Aplasia cutis congenita Correct Choice

C. Congenital syphilis

D. Keratosis Follicularis Spinulosa Decalvans

E. Morphea en coup de sabre
Aplasia cutis congenita often affects the vertex of the scalp and heals with a circular atrophic
alopecic scar


64) Splinter hemorrhage of the nail can be seen with which of the following parasitic infections?

A. Gnathostomiasis

B. Dracunculiasis

C. Trichinosis Correct Choice

D. Scabies

E. Sparganosis
Trichinosis can cause splinter hemorrhage of the nails.


65) What is the gene defect that causes red hair?
A. Melanocortin 1 receptorCorrect Choice

B. P protein

C. Tyrosine aminotransferase

D. Tyrosinase

E. Tyrosinase-related protein 1
Defects in melanocortin 1 receptor lead to the phenotypic appearance of red hair. Individuals with
red hair have increased ratio of pheomelanin to eumelanin and are at increased risk for melanoma


66) Chronic paronychia is most commonly caused by which of the following organisms?

A. Staphylococcus aureus

B. Pseudomonas aeruginosa

C. Candida albicans Correct Choice

D. Candida tropicalis

E. Trichophyton rubrum
Candida albicans is the most common pathogen associated with chronic paronychia


67) The common culprit of proximal white subungual onychomycosis is :

A. Fusarium spp

B. Candida albicans

C. Scydalidium dimidiatum

D. Trichophyton rubrumCorrect Choice

E. Trichophyton interdigitale
T. rubrum is responsible for more than 95% of these cases.


68) The syndrome including mental and physical retardation, convulsions, episodic
unconsciousness, liver enlargement, skin lesions, and dry and brittle hair showing trichorrhexis
nodosa microscopically and fluorescing red. is:


A. Citrullinemia

B. Argininosuccinic aciduriaCorrect Choice

C. Crandall's syndrome

D. Bjornstad's syndrome

E. Bazex Follicular Atrophoderma
Argininosuccinic aciduria has the features listed including trichorrhexis nodosa and red fluorescence
of the hair. Bazex's follicular atrophoderma and Crandall syndrome both have findings of pili torti
and citrullinemia has trichorrhexis nodosa. None of the other options have red fluorescence as a
feature


69) The epithelium that lies on the volar surface of the digit is the:

A. Hyponychium Correct Choice
B. Eponychium

C. Nail plate

D. Nail bed

E. Nail matrix
The epithelium that lies on the volar surface of the digit is the hyponychium. The nail bed is the
actual nail made up of tightly packed onychocytes. The nail matrix is the epithelium which starts
mid distal phalanx which generated the nail plate. It keratinizes without a granular layer and
determines the thickness of the nail plate. The nail bed is the thin epithelium immeduately beneath
the nail plate


70) Triangular lunulae are seen in what disorder?

A. Gorlin’s syndrome

B. Dyskeratosis congentia

C. Nail-patella syndrome Correct Choice

D. Neurofibromatosis II

E. Papillon-Lefevre syndrome
Triangular lunulae are seen in nail-patella syndrome which is characterized by mutation in LMX1b


71) Which of the following are characteristics of Trichorhinophalangeal syndrome?

A. Brittle hair, short stature, cerebellar ataxia

B. Shortened phalanges, sparse hair, bulbous noseCorrect Choice

C. Kinky hair, frontal bossing, small widely spaced teeth with poor enamel

D. Sparse fine hair, Short extremities, Immunodeficiency

E. Palmoplantar keratoderma, trichorrhexis nodosa, sinus infections
Characteristic features of trichorhinophalangeal syndrome are shortened phalanges, sparse hair,
and bulbous nose


72) Blue lunulae are associated with each of the following except:

A. Wilson’s disease

B. Phenolphthalein

C. Argyria

D. Quinacrine

E. Cardiac failureCorrect Choice
Blue lunulae have been reported in association with argyria, Wilson’s disease, hereditary acrolabial
telangectasia, paronychia, phenolphthalein, quinacrine, topical bichloride, and mercury exposure

73) The best test for Cushing syndrome is:

A. Plasma LH
B. Plasma DHEA
C. Plasma prolactin
D. Overnight dexamethasone suppresion testCorrect Choice
E. Plasma testosterone
The overnight dexamethasone suppression test is the test of choice for diagnosis of Cushing
syndrome. In this test, a dose of dexamethasone is given at 11pm and the plasma cortisol level is
drawn the following morning. If the Cushing syndrome is from an adrenal tumor or an ectopic ACTH
producing tumor, there is no change in cortisol secretion. A normal response is suppression of the
cortisol level by the extra dexamethasone. If the Cushings is from a pituitary tumor, at low doses of
dexamethasone there are no changes in cortisol levels while at high doses of dexamethasone there
will be a normal suppression. The other listed items are not used for diagnosis of Cushing
syndrome.


74) Which part of the nail complex results in nail pits?

A. Proximal nail fold

B. Nail bed

C. Proximal nail matrixCorrect Choice

D. Nail plate

E. Dorsal nail matrix
Pitting of the nails are punctate depressions that migrate distally and result from defect in the
proximal nail matrix. It may be seen in conditions such as psoriasis, eczema and alopecia areata


75) Spotted red lunulae are absent in which of the following conditions?

A. Rheumatoid arthritis

B. Systemic lupus erythematosus

C. Keratosis follicularisCorrect Choice

D. Lichen planus

E. Alopecia areata
Keratosis follicularis has nail findings of v-shaped nicking of the distal nail, subungual
hyperkeratosis and red/white alternating longitudinal bands. In addition to those listed above,
psoriasis can also cause spotted red lunulae


76) A patient with sparse hair, a pear-shaped broad nose and cone-shaped epiphyses likely suffers
which of the following conditions?


A. Hay-Wells Syndrome

B. Focal Dermal Hypoplasia (Goltz syndrome)

C. Trichodentoosseous Syndrome

D. Tricho-rhino-phalangeal Syndrome Correct Choice

E. Ectrodactyly-Ectodermal dysplasia-Clefting (EEC)
Tricho-rhino-phalangeal Syndrome is characterized by sparse hair, a pear-shaped broad nose and
cone-shaped epiphyses


77) A patient with diffuse hair loss developing after a thallium scan likely has which of the following
conditions?


A. Loose anagen syndrome
B. Telogen effluvium

C. Anagen effluvium Correct Choice

D. Uncombable hair syndrome

E. Catagen effluvium
Anagen effluvium can result from infusions of thallium


78) Yellow nail syndrome has been associated with use of which of the following treatments for
Rheumatoid Arthritis?


A. Infliximab

B. D-penicillamineCorrect Choice

C. Adalimumab

D. Methotrexate

E. Etanercept
D-penicillamine has been associated with the development of yellow nail syndrome in patiens with
rheumatoid arthritis. The other options are used to treat RA, but have not been associated with
development of this syndrome


79) Blue lunulae are characteristic of which disease?

A. Clubbing

B. Rubenstein-Taybi syndrome

C. Yellow nail syndrome

D. Wilson's disease Correct Choice

E. Hypertrophic pulmonary osteoarthropathy
Wilson's disease is associated with characteristic blue lunulae


80) Which genodermatosis characterized by broad thumbs and this nail disorder pictured below?

A. Rubenstein-Taybi syndrome Correct Choice

B. Proteus syndrome

C. Tuberous sclerosis

D. Rothmund-Thompson

E. Pachyonychia congenita
Rubensteom-Taybi syndrome is characterized by broad thumbs and broad nails or brachyonychia.
This syndrome is characterized by mutations in CREB-binding protein


81) What is the common cause of the “one hand-two-foot” syndrome?

A. Trichophyton rubrumCorrect Choice

B. Trichophyton mentagrophytes ( var. Interdigitale)

C. Candida albicans
D. Scytalidium dimidiatum

E. Scopulariopsis brevicaulis
This condition is due to T. rubrum in 90% of the cases , T. interdigitale and Scytalidium dimidiatum
share the 10% left


82) Regarding androgens in women, which of the following statements is NOT correct?

A. Eyebrows, eyelashes and vellus hairs are androgen-dependentCorrect Choice

B. There are no differences in eyelashes, eyebrows and vellus hair-bearing areas in men and women

C. Testosterone binds the androgen receptor

D. The hair follicle requires conversion of testosterone to dihydrotestosterone for expression of
androgen action

E. Dihydrotestosterone binds the androgen receptor
Only testosterone and dihydrotestosterone bind the androgen receptor, Eyebrows, eyelashes and
vellus hairs are NOT androgen-dependent, thus there are no differences between these areas in
men and women, and the hair follicle requires conversion of testosterone to dihydrotestosterone for
expression of androgen action


83) Ventral pterygium pictured below is characteristically associated with what disorder?

A. Chronic GVHD

B. Cicatricial pemphigoid

C. SJS/TEN

D. Lichen planus

E. Systemic sclerosisCorrect Choice
Ventral pterygium is most characteristically associated with systemic sclerosis. All other choices are
associated with dorsal pterygium


84) Arsenical poisoning is associated with what nail finding?

A. Oil spots

B. Onycholysis

C. Muehrcke's lines

D. Mee's lines Correct Choice

E. Hapalonychia
Arsenical poisoning is characteristically associated with Mee's lines


85) The hair finding characterized by an invagination of the distal hair shaft into the cup formed by
the proximal hair shaft is:


A. Pili torti

B. Pili triangulati

C. Trichoschisis

D. Trichorrhexis invaginataCorrect Choice
E. Trichorrhexis nodosa
The above description is that of trichorrhexis invaginata, which is characteristic of Netherton
syndrome. Other findings include: ichthyosis linearis circumflexa and atopy. It is caused by the
mutation of the serine protease inhibitor, SPINK5


86) Psoriatic onycholysis is caused by psoriasis in which of the following nail subunits?

A. Proximal nail fold

B. Hyponychium

C. Nail plate

D. Nail bed Correct Choice

E. Nail matrix
Nail bed involvement by psoriasis often causes onycholysis


87) A patient with synophrys, dystopia canthorum, and heterochromia irides likely has which of the
following hair abnormalities?


A. Trichorrhexis invaginata

B. Scarring alopecia

C. White forelock Correct Choice

D. Diffuse thinning

E. Trichorrhexis nodosa
White forelock is a common feature of Waardenburg syndrome


88) Mee's lines are:

A. double white transverse lines from abnormal vascular bed

B. local or diffuse hyperkeratotic tissue that develops on the lateral or proximal nail folds

C. transverse white lines that affect all nails, grow out with nail growthCorrect Choice

D. vertical black lines on a single or multiple nails

E. brownish macules beneath the nail plate
Mee's lines are transverse white lines that affect all nails, grow out with nail growth. They are seen
in arsenic poisoning, rheumatic fever, congenital heart failure, leprosy and with significant systemic
disease. Brownish macules beneath the nail plate are oil spots, seen in psoriasis. Onychophosis is
the local or diffuse hyperkeratotic tissue that develops on the lateral or proximal nail folds. The
double white transverse lines from abnormal vascular bed are Muehrcke's lines which are caused by
a nephrotic syndrome, low albumin, liver disease or malnutrition


89) Highest graft survival in hair transplantation is achieved through the use of:

A. Micrografts

B. Follicular unitsCorrect Choice

C. Single hairs

D. Minigrafts
E. 4-5 mm plugs
Seager et. al examined the survival rate of single hair grafts and compared them to follicular unit
grafts in a single patient. They found that there was a much higher survival rate in the follicular unit
grafts. They hypothesized that the extra tissue surrounding the follicular unit grafts protected them
from crush injury


90) This characteristic nail finding is seen in what disorder?

A. Darier's diseaseCorrect Choice

B. Coffin-Siris syndrome

C. Lichen planus

D. COIF syndrome

E. Pachyonychia congenita
Alternating red and white longitudinal bands on the nail are characteristic of Darier’s disease. Other
nail findings include subungual hyperkeratosis and distal v-shaped nicking


91) A commonly known cause of splinter hemorrhages in the nail is endocarditis. Which of the
following would be unlikely to cause splinter hemorrhages?


A. Trichinosis

B. Vasculitis

C. Rheumatoid arthritisCorrect Choice

D. Trauma

E. Psoriasis
Rheumatoid arthritis is not a cause of splinter hemorrhages of the nail. Endocarditis is the most
commonly thought of cause of splinter hemorrhages. Trichinosis, trauma, psoriasis and vasculitis
also can be causes of this also and should be considered when splinter hemorrhages are seen


92) The combination of ankyloblepharon, ectodermal dysplasia, and cleft palate with wiry sparse
hair is characteristic of which of the following syndromes?


A. Tricho-rhino-phalangeal Syndrome

B. Focal Dermal Hypoplasia (Goltz syndrome)

C. Trichodentoosseous Syndrome

D. Ectrodactyly-Ectodermal dysplasia-Clefting (EEC)

E. Hay-Wells Syndrome Correct Choice
Hay-Wells Syndrome, also known as AEC (Ankyloblepharon-Ectodermal dysplasia-Clefting) is
characterized by wiry, sparse hair or alopecia, ankyloblepharon, PPK, partial anhidrosis, cleft lip,
palate, absent, and dystrophic nails


93) Distal subungual onycholysis associated with paronychia is due to:

A. Scytalidium dimidiatumCorrect Choice

B. Fusarium spp

C. Trichophyton mentagrophytes
D. Candida spp

E. Trichophyton rubrum
This mold is the only one responsible for paronychia in DLSO and is more often observed in finger
than in toenails


94) A subungual hematoma covering 50% of the nailbed should be managed by which of the
following?


A. Trephination of the nail plate

B. Needle aspiration of the hematoma

C. No treatment

D. Pressure dressing

E. Removal of the nail plate Correct Choice
Removal of the nail plate is necessary for management of hematomas covering more than 25% of
the nailbed


95) Terry's nails are seen in which of the following conditions?

A. Pulmonary fibrosis

B. Bladder carcinoma

C. Renal failure

D. Aerodigestive carcinoma

E. DiabetesCorrect Choice
Terry's nails are described as all but the distal 2mm of each nail evenly white due to a defect in the
nail bed. It can be seen in cirrhosis, congenital heart failure and diabetes


96) A patient with a history of gastrointestinal polyposis presents with alopecia, generalized
pigmentation, and nail dystrophy. Which of the following is the most likely diagnosis?


A. Cronkhite-Canada Syndrome Correct Choice

B. Cowden syndrome

C. Peutz-Jegher syndrome

D. Dyskeratosis congenita

E. Gardner syndrome
Cronkhite-Canada Syndrome is a sporadic gastrointestinal polyposis syndrome characterized by
alopecia, generalized pigmentation, and nail dystrophy


97) What is the most common hair finding in Netherton's syndrome?

A. Trichorrhexis invaginataCorrect Choice

B. Pili torti

C. Monilethrix

D. Trichoschisis
E. Trichorrhexis nodosa
Netherton syndrome is an an autosomal recessive genodermatosis that is caused by a mutation in
SPINK5. The characteristic findings include trichorrhexis invaginata, ichthyosis linearis circumflexa,
and food allergy. Infants may present with erythroderma


98) Transverse white bands on one or two nails is caused by:

A. trauma to the matrixCorrect Choice

B. psoriasis

C. systemic lupus erythematosis

D. alopecia areata

E. lichen planus
Longitudinal leukonychia or transverse white bands are caused by trauma to the nail matrix. AA,
psoriasis, SLE and LP all cause spotted red lunulae, not transverse white bands


99) Mutations in c-kit are associated with which of the following conditions?

A. Piebaldism Correct Choice

B. Waardenburg syndrome

C. Vogt-koyanagi-harada

D. Tuberous sclerosis

E. Cornelia de Lange Syndrome
Piebaldism is caused by mutations in c-kit


100) Which of the following medication is a cause of hirsutism without virilization?

A. DiazoxideCorrect Choice

B. Dinitrochlorobenzene

C. Dapsone

D. Diazepam

E. Dantrolene
Phenytoin, diazoxide, cyclosporine and hexachlorobenzene all can cause increased hair growth in
patients. In addition, oral (and topical) minoxidil can cause hirsutism. The other listed medications
do not cause hirsutism


101) What nutritional deficiency can lead to this nail finding?

A. Vitamin B12

B. Thiamine

C. IronCorrect Choice

D. Magnesium

E. Vitamin C
Iron deficiency can lead to spoon nail deformity or koilonychias
102) A 42 year old woman presents with the complaint of excess hair growth on her face. She has
normal menses and has recently had her "annual" exam and the note relates normal sized ovaries.
What is the most logical next step?


A. Refer her to endocrinology

B. Order a CT of the abdomen

C. Send a 21-hydroxylase enzyme deficiency test

D. Check plasma levels of androstenedione and testosteroneCorrect Choice

E. Biopsy from the most affected area
Women with idiopathic hirsutism will have evidence of androgen excess but with normal menses,
normal-sized ovaries and no evidence of tumors of adrenal or ovary and normal adrenal function.
They will often have slight elevations of plasma androstenedione and testosterone. Check the blood
levels of the plasma steroids would be a logical first step


103) Ferritin levels need to be, at minimum, above which of the serum levels to treat iron-
deficiency related telogen effluvium?


A. 30ng/dL

B. 20ng/dL

C. 60ng/dL

D. 10ng/dL

E. 40ng/dLCorrect Choice
The therapeutic target for iron deficiency related telogen effluvium is greater than 40mg/dL. Other
causes of telogen effluvium include thyroid dysfunction, as well as such medications as beta-
blockers, antihyperlipidemic drugs, and NSAIDs


104) Plummer's nails are:

A. defective keratinization on the proximal matrix

B. caused by B12 deficiency

C. hereditary or from shoe trauma

D. caused by onycholysis from hypothyroidismCorrect Choice

E. caused by cirrhosis
Plummer's nails are caused by onycholysis from hypothyroidism. Pincer nails are hereditary or from
show trauma, usually 1st toenails. Nail pitting is caused by defective keratinization on the proximal
matrix. B12 deficiency can cause nail pigmentation


105) What determines the thickness of the nail plate?

A. Nail plate

B. Hyponychium

C. Proximal nail fold

D. Nail bed
E. Nail matrixCorrect Choice
The nail matrix determines the thickness of the nail plate


106) A patient presents with 20 nails with absent cuticles and lunulae, slow growth, dystrophic
shape and a yellow hue. Which of the following findings is/are associated?


A. pleural effusionsCorrect Choice

B. cirrhosis

C. pulmonary fibrosis

D. chest pain

E. upper extremity edema
Yellow Nail syndrome has been associated with lower extremity edema, bronchiectasis and pleural
effusions. The other listed findings are not part of Yellow Nail syndrome


107) A rapid onset of hair growth with or without accompanying virilization can occur in which of
the following conditions EXCEPT:


A. Ovarian cystCorrect Choice

B. Adrenal carcinoma

C. Arrhenoblastoma

D. Krukenburg tumor of the ovary

E. Adrenal adenoma
All of these options except an ovarian cyst can cause a rapid onset of hair growth with or without
accompanying virilization. An Arrhenoblastoma is a tumor of the ovary that secretes testosterone


108) Deposition of mucin in the hair follicle can be associated with which disease?

A. Keratosis Follicularis Spinulosa Decalvans

B. Adams-Oliver Syndrome

C. Mycosis fungoides Correct Choice

D. Perifolliculitis Capitis Abscedens et Suffodiens

E. Acne Keloidalis Nuchae
Mycosis fungoides can be associated with follicular mucinosis


109) A 32 year-old woman presents with the complaint shown in the image. This finding began one
year ago and has slowly worsened. Her female relatives do not have similar hair patterns. Serum
DHEA returns at 8544 ng/ml and serum testosterone at 3 ng/ml. What is the most likely diagnosis
for cause of her hirsutism:


A. Poryphria cutanea tarda

B. Erythropoietic protoporphyria

C. An androgen producing tumorCorrect Choice

D. Stein-Leventhal syndrome

E. Acute intermittent porphyria
DHEA > 8000 ng/ml and testosterone > 2 ng/ml are suggestive of a secreting neoplasm. Porphyria
cutanea tarda and Stein-Leventhal syndrome do present with hirsutism, but will not have the
elevated levels of DHEA and testosterone in the serum. Acute intermittent porphyria and
Erythropoietic protoporphyria do not have hypertrichosis


110) Plummer's nails are associated with which of the following disorders?

A. Dermatomyositis

B. Systemic sclerosis

C. Lupus erythematosus

D. Lead poisoning

E. Hypothyroidism Correct Choice
Plummer's nails are onycholysis due to hypothyroidism


111) This hair finding is caused by mutations in what gene?

A. SERCA2

B. Keratin 16

C. Dyskerin

D. NEMO

E. SPINK 5Correct Choice
Mutations in SPINK 5, a serine protease inhibitor leads to Netherton’s syndrome which is
characterized by trichorrhexis invaginata or bamboo hair


112) Air spaces in the hair shaft lead to this condition pictured below – name the condition:

A. Wooly hair

B. Trichoschisis

C. Pohl Pinkus constriction

D. Pili annulatiCorrect Choice

E. Pili trianguli et canaliculi
Pili anulati is caused by air spaces in the hair shaft.


113) Thin hair with premature graying is characteristic of which of the following syndromes?

A. Cornelia de Lange Syndrome

B. Noonan Syndrome

C. Werner syndrome Correct Choice

D. Waardenburg's syndrome

E. Turner Syndrome
Explanation


114) Mutations in hair keratins hHB6 and hHB1 cause this characteristic finding pictured below –
name the condition:
A. Trichoschisis

B. Trichoptilosis

C. MonilethrixCorrect Choice

D. Hair casts

E. Tiger-tail hair
Mutations in the hair keratins, hHB6 and hHB 1 cause monilethrix.


115) A patient with psoriasis has pitting of the nails. This finding is due to involvement of which
part of the nail unit?


A. Distal matrix

B. Proximal matrixCorrect Choice

C. Hyponychium

D. Proximal nail fold

E. Nail bed
Disease in the proximal matrix is responsible for producing pitting, onychorrhexis, and Beau’s lines.
Changes in the intermediate matrix can cause leukonychia, and the distal matrix may be
responsible for focal onycholysis, thinned nail plate, and erythema of the lunula.


116) Björnstad syndrome is the combination of pili torti and which of the following features?

A. Lentigines

B. Deafness Correct Choice

C. Hypogonadism

D. White forelock

E. Seizures
Deafness and pili torti are the two cardinal features of Björnstad syndrome

Weitere ähnliche Inhalte

Was ist angesagt?

ETAS_MCQ_12 bullous diseases and blistering
ETAS_MCQ_12 bullous diseases and blisteringETAS_MCQ_12 bullous diseases and blistering
ETAS_MCQ_12 bullous diseases and blisteringDerma202
 
ETAS_MCQ_13 photobiology and photosensitivity disorders
ETAS_MCQ_13 photobiology and photosensitivity disordersETAS_MCQ_13 photobiology and photosensitivity disorders
ETAS_MCQ_13 photobiology and photosensitivity disordersDerma202
 
ETAS_MCQ_15 dermatologic and cosmetic surgery
ETAS_MCQ_15 dermatologic and cosmetic surgeryETAS_MCQ_15 dermatologic and cosmetic surgery
ETAS_MCQ_15 dermatologic and cosmetic surgeryDerma202
 
ETAS_MCQ_01 structures of skin
ETAS_MCQ_01 structures of skinETAS_MCQ_01 structures of skin
ETAS_MCQ_01 structures of skinDerma202
 
Previous year question on pemphigus vulgaris based on neet pg, usmle, plab an...
Previous year question on pemphigus vulgaris based on neet pg, usmle, plab an...Previous year question on pemphigus vulgaris based on neet pg, usmle, plab an...
Previous year question on pemphigus vulgaris based on neet pg, usmle, plab an...Medico Apps
 
Previous year question on lichen planus based on neet pg, usmle, plab and fmg...
Previous year question on lichen planus based on neet pg, usmle, plab and fmg...Previous year question on lichen planus based on neet pg, usmle, plab and fmg...
Previous year question on lichen planus based on neet pg, usmle, plab and fmg...Abhishek Gupta
 
ETAS_MCQ_02 immunodermatology
ETAS_MCQ_02 immunodermatologyETAS_MCQ_02 immunodermatology
ETAS_MCQ_02 immunodermatologyDerma202
 
Dermatology board review
Dermatology board reviewDermatology board review
Dermatology board reviewAhmed Amer
 
Internal Medicine Image Challenge MCQs
Internal Medicine Image Challenge MCQsInternal Medicine Image Challenge MCQs
Internal Medicine Image Challenge MCQsSherif Elbadrawy
 
Previous year question on tonsils based on neet pg, usmle, plab and fmge or m...
Previous year question on tonsils based on neet pg, usmle, plab and fmge or m...Previous year question on tonsils based on neet pg, usmle, plab and fmge or m...
Previous year question on tonsils based on neet pg, usmle, plab and fmge or m...Abhishek Gupta
 
Vesiculobullous II
Vesiculobullous IIVesiculobullous II
Vesiculobullous IIIAU Dent
 
Vohwinkel Syndrome vs. KID Syndrome
Vohwinkel Syndrome vs. KID Syndrome Vohwinkel Syndrome vs. KID Syndrome
Vohwinkel Syndrome vs. KID Syndrome JenniferArmstrong6
 
Previous year question on polio based on neet pg, usmle, plab and fmge or mci...
Previous year question on polio based on neet pg, usmle, plab and fmge or mci...Previous year question on polio based on neet pg, usmle, plab and fmge or mci...
Previous year question on polio based on neet pg, usmle, plab and fmge or mci...Abhishek Gupta
 
Vitiligo in association with Erythema dyschromicum perstans
Vitiligo in association with Erythema dyschromicum perstansVitiligo in association with Erythema dyschromicum perstans
Vitiligo in association with Erythema dyschromicum perstansVR Foundation
 

Was ist angesagt? (20)

ETAS_MCQ_12 bullous diseases and blistering
ETAS_MCQ_12 bullous diseases and blisteringETAS_MCQ_12 bullous diseases and blistering
ETAS_MCQ_12 bullous diseases and blistering
 
Derma.
Derma.Derma.
Derma.
 
ETAS_MCQ_13 photobiology and photosensitivity disorders
ETAS_MCQ_13 photobiology and photosensitivity disordersETAS_MCQ_13 photobiology and photosensitivity disorders
ETAS_MCQ_13 photobiology and photosensitivity disorders
 
ETAS_MCQ_15 dermatologic and cosmetic surgery
ETAS_MCQ_15 dermatologic and cosmetic surgeryETAS_MCQ_15 dermatologic and cosmetic surgery
ETAS_MCQ_15 dermatologic and cosmetic surgery
 
Ganyang MCQ Dermatology
Ganyang MCQ DermatologyGanyang MCQ Dermatology
Ganyang MCQ Dermatology
 
ETAS_MCQ_01 structures of skin
ETAS_MCQ_01 structures of skinETAS_MCQ_01 structures of skin
ETAS_MCQ_01 structures of skin
 
Previous year question on pemphigus vulgaris based on neet pg, usmle, plab an...
Previous year question on pemphigus vulgaris based on neet pg, usmle, plab an...Previous year question on pemphigus vulgaris based on neet pg, usmle, plab an...
Previous year question on pemphigus vulgaris based on neet pg, usmle, plab an...
 
Dermatology for MRCP
Dermatology for MRCPDermatology for MRCP
Dermatology for MRCP
 
Previous year question on lichen planus based on neet pg, usmle, plab and fmg...
Previous year question on lichen planus based on neet pg, usmle, plab and fmg...Previous year question on lichen planus based on neet pg, usmle, plab and fmg...
Previous year question on lichen planus based on neet pg, usmle, plab and fmg...
 
Mrcp Part 2 Witten Exam
Mrcp Part 2 Witten ExamMrcp Part 2 Witten Exam
Mrcp Part 2 Witten Exam
 
ETAS_MCQ_02 immunodermatology
ETAS_MCQ_02 immunodermatologyETAS_MCQ_02 immunodermatology
ETAS_MCQ_02 immunodermatology
 
Dermatology board review
Dermatology board reviewDermatology board review
Dermatology board review
 
Mrcp 2 dermatology
Mrcp 2 dermatologyMrcp 2 dermatology
Mrcp 2 dermatology
 
Internal Medicine Image Challenge MCQs
Internal Medicine Image Challenge MCQsInternal Medicine Image Challenge MCQs
Internal Medicine Image Challenge MCQs
 
Previous year question on tonsils based on neet pg, usmle, plab and fmge or m...
Previous year question on tonsils based on neet pg, usmle, plab and fmge or m...Previous year question on tonsils based on neet pg, usmle, plab and fmge or m...
Previous year question on tonsils based on neet pg, usmle, plab and fmge or m...
 
Vesiculobullous II
Vesiculobullous IIVesiculobullous II
Vesiculobullous II
 
Vohwinkel Syndrome vs. KID Syndrome
Vohwinkel Syndrome vs. KID Syndrome Vohwinkel Syndrome vs. KID Syndrome
Vohwinkel Syndrome vs. KID Syndrome
 
Annular erythema
Annular erythemaAnnular erythema
Annular erythema
 
Previous year question on polio based on neet pg, usmle, plab and fmge or mci...
Previous year question on polio based on neet pg, usmle, plab and fmge or mci...Previous year question on polio based on neet pg, usmle, plab and fmge or mci...
Previous year question on polio based on neet pg, usmle, plab and fmge or mci...
 
Vitiligo in association with Erythema dyschromicum perstans
Vitiligo in association with Erythema dyschromicum perstansVitiligo in association with Erythema dyschromicum perstans
Vitiligo in association with Erythema dyschromicum perstans
 

Ähnlich wie Hair and nail disorders guide

Mohamed's oral pathology,medicine and radiography
Mohamed's oral pathology,medicine and radiographyMohamed's oral pathology,medicine and radiography
Mohamed's oral pathology,medicine and radiographyRosa Martinez
 
Superficial fungal skin infections.ppt
Superficial fungal skin infections.pptSuperficial fungal skin infections.ppt
Superficial fungal skin infections.pptaliimad10
 
Krok 1 - 2008 Question Paper (Stomatology)
Krok 1 - 2008 Question Paper (Stomatology)Krok 1 - 2008 Question Paper (Stomatology)
Krok 1 - 2008 Question Paper (Stomatology)Eneutron
 
Introduction to Myco and viro.pptx viruses and microb
Introduction to Myco and viro.pptx viruses and microbIntroduction to Myco and viro.pptx viruses and microb
Introduction to Myco and viro.pptx viruses and microbNathanDanielgashahun
 
DERMATOMYCOSIS Epidermophyton KKR.pptx
DERMATOMYCOSIS Epidermophyton   KKR.pptxDERMATOMYCOSIS Epidermophyton   KKR.pptx
DERMATOMYCOSIS Epidermophyton KKR.pptxKARTHIK REDDY C A
 
Krok 1 - 2006 Question Paper (General medicine)
Krok 1 - 2006 Question Paper (General medicine)Krok 1 - 2006 Question Paper (General medicine)
Krok 1 - 2006 Question Paper (General medicine)Eneutron
 
Diseases of External Ear.pptx
Diseases of External Ear.pptxDiseases of External Ear.pptx
Diseases of External Ear.pptxSUMEET578391
 
Dermatophyte infection (2).pptx
Dermatophyte infection (2).pptxDermatophyte infection (2).pptx
Dermatophyte infection (2).pptxhussainAltaher
 

Ähnlich wie Hair and nail disorders guide (20)

Mohamed's oral pathology,medicine and radiography
Mohamed's oral pathology,medicine and radiographyMohamed's oral pathology,medicine and radiography
Mohamed's oral pathology,medicine and radiography
 
Superficial fungal skin infections.ppt
Superficial fungal skin infections.pptSuperficial fungal skin infections.ppt
Superficial fungal skin infections.ppt
 
Krok 1 - 2008 Question Paper (Stomatology)
Krok 1 - 2008 Question Paper (Stomatology)Krok 1 - 2008 Question Paper (Stomatology)
Krok 1 - 2008 Question Paper (Stomatology)
 
Introduction to Myco and viro.pptx viruses and microb
Introduction to Myco and viro.pptx viruses and microbIntroduction to Myco and viro.pptx viruses and microb
Introduction to Myco and viro.pptx viruses and microb
 
CSOM
CSOMCSOM
CSOM
 
Onicomicosis
OnicomicosisOnicomicosis
Onicomicosis
 
7. Fungal infections
7. Fungal infections7. Fungal infections
7. Fungal infections
 
DERMATOMYCOSIS Epidermophyton KKR.pptx
DERMATOMYCOSIS Epidermophyton   KKR.pptxDERMATOMYCOSIS Epidermophyton   KKR.pptx
DERMATOMYCOSIS Epidermophyton KKR.pptx
 
Dermatomycosis
Dermatomycosis Dermatomycosis
Dermatomycosis
 
diseasesofexternalear
diseasesofexternaleardiseasesofexternalear
diseasesofexternalear
 
Mcqs for Ophthal pgs 1
Mcqs  for Ophthal pgs 1Mcqs  for Ophthal pgs 1
Mcqs for Ophthal pgs 1
 
Krok 1 - 2006 Question Paper (General medicine)
Krok 1 - 2006 Question Paper (General medicine)Krok 1 - 2006 Question Paper (General medicine)
Krok 1 - 2006 Question Paper (General medicine)
 
2) aetiology & epidemiology of com
2) aetiology & epidemiology of com2) aetiology & epidemiology of com
2) aetiology & epidemiology of com
 
Dermatology 5th year, 5th lecture (Dr. Darseem)
Dermatology 5th year, 5th lecture (Dr. Darseem)Dermatology 5th year, 5th lecture (Dr. Darseem)
Dermatology 5th year, 5th lecture (Dr. Darseem)
 
Diseases of External Ear.pptx
Diseases of External Ear.pptxDiseases of External Ear.pptx
Diseases of External Ear.pptx
 
Otitis media
Otitis mediaOtitis media
Otitis media
 
Rao Abu Bakar 2017009.docx
Rao Abu Bakar 2017009.docxRao Abu Bakar 2017009.docx
Rao Abu Bakar 2017009.docx
 
Dermatology 5th year, 2nd lecture (Dr. Darseem)
Dermatology 5th year, 2nd lecture (Dr. Darseem)Dermatology 5th year, 2nd lecture (Dr. Darseem)
Dermatology 5th year, 2nd lecture (Dr. Darseem)
 
Dermatophyte infection (2).pptx
Dermatophyte infection (2).pptxDermatophyte infection (2).pptx
Dermatophyte infection (2).pptx
 
ENT - Sinusitis.pptx
ENT - Sinusitis.pptxENT - Sinusitis.pptx
ENT - Sinusitis.pptx
 

Mehr von Derma202

Phototherapy treatment protocol
Phototherapy treatment protocolPhototherapy treatment protocol
Phototherapy treatment protocolDerma202
 
Histopathplogical photos
Histopathplogical photosHistopathplogical photos
Histopathplogical photosDerma202
 
Slide study from ETAS
Slide  study from ETASSlide  study from ETAS
Slide study from ETASDerma202
 
Arab board primary exam in dermatology 2012
Arab board primary exam  in dermatology 2012Arab board primary exam  in dermatology 2012
Arab board primary exam in dermatology 2012Derma202
 
Dermatology
DermatologyDermatology
DermatologyDerma202
 
Derm handbook for medical students and junior doctors 2010
Derm handbook for medical students and junior doctors 2010Derm handbook for medical students and junior doctors 2010
Derm handbook for medical students and junior doctors 2010Derma202
 

Mehr von Derma202 (6)

Phototherapy treatment protocol
Phototherapy treatment protocolPhototherapy treatment protocol
Phototherapy treatment protocol
 
Histopathplogical photos
Histopathplogical photosHistopathplogical photos
Histopathplogical photos
 
Slide study from ETAS
Slide  study from ETASSlide  study from ETAS
Slide study from ETAS
 
Arab board primary exam in dermatology 2012
Arab board primary exam  in dermatology 2012Arab board primary exam  in dermatology 2012
Arab board primary exam in dermatology 2012
 
Dermatology
DermatologyDermatology
Dermatology
 
Derm handbook for medical students and junior doctors 2010
Derm handbook for medical students and junior doctors 2010Derm handbook for medical students and junior doctors 2010
Derm handbook for medical students and junior doctors 2010
 

Kürzlich hochgeladen

Call Girls Kochi Just Call 9907093804 Top Class Call Girl Service Available
Call Girls Kochi Just Call 9907093804 Top Class Call Girl Service AvailableCall Girls Kochi Just Call 9907093804 Top Class Call Girl Service Available
Call Girls Kochi Just Call 9907093804 Top Class Call Girl Service AvailableDipal Arora
 
Night 7k to 12k Navi Mumbai Call Girl Photo 👉 BOOK NOW 9833363713 👈 ♀️ night ...
Night 7k to 12k Navi Mumbai Call Girl Photo 👉 BOOK NOW 9833363713 👈 ♀️ night ...Night 7k to 12k Navi Mumbai Call Girl Photo 👉 BOOK NOW 9833363713 👈 ♀️ night ...
Night 7k to 12k Navi Mumbai Call Girl Photo 👉 BOOK NOW 9833363713 👈 ♀️ night ...aartirawatdelhi
 
Call Girls Aurangabad Just Call 9907093804 Top Class Call Girl Service Available
Call Girls Aurangabad Just Call 9907093804 Top Class Call Girl Service AvailableCall Girls Aurangabad Just Call 9907093804 Top Class Call Girl Service Available
Call Girls Aurangabad Just Call 9907093804 Top Class Call Girl Service AvailableDipal Arora
 
Call Girls Coimbatore Just Call 9907093804 Top Class Call Girl Service Available
Call Girls Coimbatore Just Call 9907093804 Top Class Call Girl Service AvailableCall Girls Coimbatore Just Call 9907093804 Top Class Call Girl Service Available
Call Girls Coimbatore Just Call 9907093804 Top Class Call Girl Service AvailableDipal Arora
 
Call Girls Jabalpur Just Call 9907093804 Top Class Call Girl Service Available
Call Girls Jabalpur Just Call 9907093804 Top Class Call Girl Service AvailableCall Girls Jabalpur Just Call 9907093804 Top Class Call Girl Service Available
Call Girls Jabalpur Just Call 9907093804 Top Class Call Girl Service AvailableDipal Arora
 
Call Girls Bareilly Just Call 9907093804 Top Class Call Girl Service Available
Call Girls Bareilly Just Call 9907093804 Top Class Call Girl Service AvailableCall Girls Bareilly Just Call 9907093804 Top Class Call Girl Service Available
Call Girls Bareilly Just Call 9907093804 Top Class Call Girl Service AvailableDipal Arora
 
Top Rated Bangalore Call Girls Richmond Circle ⟟ 8250192130 ⟟ Call Me For Gen...
Top Rated Bangalore Call Girls Richmond Circle ⟟ 8250192130 ⟟ Call Me For Gen...Top Rated Bangalore Call Girls Richmond Circle ⟟ 8250192130 ⟟ Call Me For Gen...
Top Rated Bangalore Call Girls Richmond Circle ⟟ 8250192130 ⟟ Call Me For Gen...narwatsonia7
 
VIP Mumbai Call Girls Hiranandani Gardens Just Call 9920874524 with A/C Room ...
VIP Mumbai Call Girls Hiranandani Gardens Just Call 9920874524 with A/C Room ...VIP Mumbai Call Girls Hiranandani Gardens Just Call 9920874524 with A/C Room ...
VIP Mumbai Call Girls Hiranandani Gardens Just Call 9920874524 with A/C Room ...Garima Khatri
 
💎VVIP Kolkata Call Girls Parganas🩱7001035870🩱Independent Girl ( Ac Rooms Avai...
💎VVIP Kolkata Call Girls Parganas🩱7001035870🩱Independent Girl ( Ac Rooms Avai...💎VVIP Kolkata Call Girls Parganas🩱7001035870🩱Independent Girl ( Ac Rooms Avai...
💎VVIP Kolkata Call Girls Parganas🩱7001035870🩱Independent Girl ( Ac Rooms Avai...Taniya Sharma
 
Call Girls Siliguri Just Call 9907093804 Top Class Call Girl Service Available
Call Girls Siliguri Just Call 9907093804 Top Class Call Girl Service AvailableCall Girls Siliguri Just Call 9907093804 Top Class Call Girl Service Available
Call Girls Siliguri Just Call 9907093804 Top Class Call Girl Service AvailableDipal Arora
 
Lucknow Call girls - 8800925952 - 24x7 service with hotel room
Lucknow Call girls - 8800925952 - 24x7 service with hotel roomLucknow Call girls - 8800925952 - 24x7 service with hotel room
Lucknow Call girls - 8800925952 - 24x7 service with hotel roomdiscovermytutordmt
 
Call Girls Varanasi Just Call 9907093804 Top Class Call Girl Service Available
Call Girls Varanasi Just Call 9907093804 Top Class Call Girl Service AvailableCall Girls Varanasi Just Call 9907093804 Top Class Call Girl Service Available
Call Girls Varanasi Just Call 9907093804 Top Class Call Girl Service AvailableDipal Arora
 
The Most Attractive Hyderabad Call Girls Kothapet 𖠋 6297143586 𖠋 Will You Mis...
The Most Attractive Hyderabad Call Girls Kothapet 𖠋 6297143586 𖠋 Will You Mis...The Most Attractive Hyderabad Call Girls Kothapet 𖠋 6297143586 𖠋 Will You Mis...
The Most Attractive Hyderabad Call Girls Kothapet 𖠋 6297143586 𖠋 Will You Mis...chandars293
 
Top Rated Hyderabad Call Girls Erragadda ⟟ 6297143586 ⟟ Call Me For Genuine ...
Top Rated  Hyderabad Call Girls Erragadda ⟟ 6297143586 ⟟ Call Me For Genuine ...Top Rated  Hyderabad Call Girls Erragadda ⟟ 6297143586 ⟟ Call Me For Genuine ...
Top Rated Hyderabad Call Girls Erragadda ⟟ 6297143586 ⟟ Call Me For Genuine ...chandars293
 
Call Girls Nagpur Just Call 9907093804 Top Class Call Girl Service Available
Call Girls Nagpur Just Call 9907093804 Top Class Call Girl Service AvailableCall Girls Nagpur Just Call 9907093804 Top Class Call Girl Service Available
Call Girls Nagpur Just Call 9907093804 Top Class Call Girl Service AvailableDipal Arora
 
Russian Escorts Girls Nehru Place ZINATHI 🔝9711199012 ☪ 24/7 Call Girls Delhi
Russian Escorts Girls  Nehru Place ZINATHI 🔝9711199012 ☪ 24/7 Call Girls DelhiRussian Escorts Girls  Nehru Place ZINATHI 🔝9711199012 ☪ 24/7 Call Girls Delhi
Russian Escorts Girls Nehru Place ZINATHI 🔝9711199012 ☪ 24/7 Call Girls DelhiAlinaDevecerski
 
All Time Service Available Call Girls Marine Drive 📳 9820252231 For 18+ VIP C...
All Time Service Available Call Girls Marine Drive 📳 9820252231 For 18+ VIP C...All Time Service Available Call Girls Marine Drive 📳 9820252231 For 18+ VIP C...
All Time Service Available Call Girls Marine Drive 📳 9820252231 For 18+ VIP C...Arohi Goyal
 
(Rocky) Jaipur Call Girl - 9521753030 Escorts Service 50% Off with Cash ON De...
(Rocky) Jaipur Call Girl - 9521753030 Escorts Service 50% Off with Cash ON De...(Rocky) Jaipur Call Girl - 9521753030 Escorts Service 50% Off with Cash ON De...
(Rocky) Jaipur Call Girl - 9521753030 Escorts Service 50% Off with Cash ON De...indiancallgirl4rent
 
Bangalore Call Girl Whatsapp Number 100% Complete Your Sexual Needs
Bangalore Call Girl Whatsapp Number 100% Complete Your Sexual NeedsBangalore Call Girl Whatsapp Number 100% Complete Your Sexual Needs
Bangalore Call Girl Whatsapp Number 100% Complete Your Sexual NeedsGfnyt
 
Vip Call Girls Anna Salai Chennai 👉 8250192130 ❣️💯 Top Class Girls Available
Vip Call Girls Anna Salai Chennai 👉 8250192130 ❣️💯 Top Class Girls AvailableVip Call Girls Anna Salai Chennai 👉 8250192130 ❣️💯 Top Class Girls Available
Vip Call Girls Anna Salai Chennai 👉 8250192130 ❣️💯 Top Class Girls AvailableNehru place Escorts
 

Kürzlich hochgeladen (20)

Call Girls Kochi Just Call 9907093804 Top Class Call Girl Service Available
Call Girls Kochi Just Call 9907093804 Top Class Call Girl Service AvailableCall Girls Kochi Just Call 9907093804 Top Class Call Girl Service Available
Call Girls Kochi Just Call 9907093804 Top Class Call Girl Service Available
 
Night 7k to 12k Navi Mumbai Call Girl Photo 👉 BOOK NOW 9833363713 👈 ♀️ night ...
Night 7k to 12k Navi Mumbai Call Girl Photo 👉 BOOK NOW 9833363713 👈 ♀️ night ...Night 7k to 12k Navi Mumbai Call Girl Photo 👉 BOOK NOW 9833363713 👈 ♀️ night ...
Night 7k to 12k Navi Mumbai Call Girl Photo 👉 BOOK NOW 9833363713 👈 ♀️ night ...
 
Call Girls Aurangabad Just Call 9907093804 Top Class Call Girl Service Available
Call Girls Aurangabad Just Call 9907093804 Top Class Call Girl Service AvailableCall Girls Aurangabad Just Call 9907093804 Top Class Call Girl Service Available
Call Girls Aurangabad Just Call 9907093804 Top Class Call Girl Service Available
 
Call Girls Coimbatore Just Call 9907093804 Top Class Call Girl Service Available
Call Girls Coimbatore Just Call 9907093804 Top Class Call Girl Service AvailableCall Girls Coimbatore Just Call 9907093804 Top Class Call Girl Service Available
Call Girls Coimbatore Just Call 9907093804 Top Class Call Girl Service Available
 
Call Girls Jabalpur Just Call 9907093804 Top Class Call Girl Service Available
Call Girls Jabalpur Just Call 9907093804 Top Class Call Girl Service AvailableCall Girls Jabalpur Just Call 9907093804 Top Class Call Girl Service Available
Call Girls Jabalpur Just Call 9907093804 Top Class Call Girl Service Available
 
Call Girls Bareilly Just Call 9907093804 Top Class Call Girl Service Available
Call Girls Bareilly Just Call 9907093804 Top Class Call Girl Service AvailableCall Girls Bareilly Just Call 9907093804 Top Class Call Girl Service Available
Call Girls Bareilly Just Call 9907093804 Top Class Call Girl Service Available
 
Top Rated Bangalore Call Girls Richmond Circle ⟟ 8250192130 ⟟ Call Me For Gen...
Top Rated Bangalore Call Girls Richmond Circle ⟟ 8250192130 ⟟ Call Me For Gen...Top Rated Bangalore Call Girls Richmond Circle ⟟ 8250192130 ⟟ Call Me For Gen...
Top Rated Bangalore Call Girls Richmond Circle ⟟ 8250192130 ⟟ Call Me For Gen...
 
VIP Mumbai Call Girls Hiranandani Gardens Just Call 9920874524 with A/C Room ...
VIP Mumbai Call Girls Hiranandani Gardens Just Call 9920874524 with A/C Room ...VIP Mumbai Call Girls Hiranandani Gardens Just Call 9920874524 with A/C Room ...
VIP Mumbai Call Girls Hiranandani Gardens Just Call 9920874524 with A/C Room ...
 
💎VVIP Kolkata Call Girls Parganas🩱7001035870🩱Independent Girl ( Ac Rooms Avai...
💎VVIP Kolkata Call Girls Parganas🩱7001035870🩱Independent Girl ( Ac Rooms Avai...💎VVIP Kolkata Call Girls Parganas🩱7001035870🩱Independent Girl ( Ac Rooms Avai...
💎VVIP Kolkata Call Girls Parganas🩱7001035870🩱Independent Girl ( Ac Rooms Avai...
 
Call Girls Siliguri Just Call 9907093804 Top Class Call Girl Service Available
Call Girls Siliguri Just Call 9907093804 Top Class Call Girl Service AvailableCall Girls Siliguri Just Call 9907093804 Top Class Call Girl Service Available
Call Girls Siliguri Just Call 9907093804 Top Class Call Girl Service Available
 
Lucknow Call girls - 8800925952 - 24x7 service with hotel room
Lucknow Call girls - 8800925952 - 24x7 service with hotel roomLucknow Call girls - 8800925952 - 24x7 service with hotel room
Lucknow Call girls - 8800925952 - 24x7 service with hotel room
 
Call Girls Varanasi Just Call 9907093804 Top Class Call Girl Service Available
Call Girls Varanasi Just Call 9907093804 Top Class Call Girl Service AvailableCall Girls Varanasi Just Call 9907093804 Top Class Call Girl Service Available
Call Girls Varanasi Just Call 9907093804 Top Class Call Girl Service Available
 
The Most Attractive Hyderabad Call Girls Kothapet 𖠋 6297143586 𖠋 Will You Mis...
The Most Attractive Hyderabad Call Girls Kothapet 𖠋 6297143586 𖠋 Will You Mis...The Most Attractive Hyderabad Call Girls Kothapet 𖠋 6297143586 𖠋 Will You Mis...
The Most Attractive Hyderabad Call Girls Kothapet 𖠋 6297143586 𖠋 Will You Mis...
 
Top Rated Hyderabad Call Girls Erragadda ⟟ 6297143586 ⟟ Call Me For Genuine ...
Top Rated  Hyderabad Call Girls Erragadda ⟟ 6297143586 ⟟ Call Me For Genuine ...Top Rated  Hyderabad Call Girls Erragadda ⟟ 6297143586 ⟟ Call Me For Genuine ...
Top Rated Hyderabad Call Girls Erragadda ⟟ 6297143586 ⟟ Call Me For Genuine ...
 
Call Girls Nagpur Just Call 9907093804 Top Class Call Girl Service Available
Call Girls Nagpur Just Call 9907093804 Top Class Call Girl Service AvailableCall Girls Nagpur Just Call 9907093804 Top Class Call Girl Service Available
Call Girls Nagpur Just Call 9907093804 Top Class Call Girl Service Available
 
Russian Escorts Girls Nehru Place ZINATHI 🔝9711199012 ☪ 24/7 Call Girls Delhi
Russian Escorts Girls  Nehru Place ZINATHI 🔝9711199012 ☪ 24/7 Call Girls DelhiRussian Escorts Girls  Nehru Place ZINATHI 🔝9711199012 ☪ 24/7 Call Girls Delhi
Russian Escorts Girls Nehru Place ZINATHI 🔝9711199012 ☪ 24/7 Call Girls Delhi
 
All Time Service Available Call Girls Marine Drive 📳 9820252231 For 18+ VIP C...
All Time Service Available Call Girls Marine Drive 📳 9820252231 For 18+ VIP C...All Time Service Available Call Girls Marine Drive 📳 9820252231 For 18+ VIP C...
All Time Service Available Call Girls Marine Drive 📳 9820252231 For 18+ VIP C...
 
(Rocky) Jaipur Call Girl - 9521753030 Escorts Service 50% Off with Cash ON De...
(Rocky) Jaipur Call Girl - 9521753030 Escorts Service 50% Off with Cash ON De...(Rocky) Jaipur Call Girl - 9521753030 Escorts Service 50% Off with Cash ON De...
(Rocky) Jaipur Call Girl - 9521753030 Escorts Service 50% Off with Cash ON De...
 
Bangalore Call Girl Whatsapp Number 100% Complete Your Sexual Needs
Bangalore Call Girl Whatsapp Number 100% Complete Your Sexual NeedsBangalore Call Girl Whatsapp Number 100% Complete Your Sexual Needs
Bangalore Call Girl Whatsapp Number 100% Complete Your Sexual Needs
 
Vip Call Girls Anna Salai Chennai 👉 8250192130 ❣️💯 Top Class Girls Available
Vip Call Girls Anna Salai Chennai 👉 8250192130 ❣️💯 Top Class Girls AvailableVip Call Girls Anna Salai Chennai 👉 8250192130 ❣️💯 Top Class Girls Available
Vip Call Girls Anna Salai Chennai 👉 8250192130 ❣️💯 Top Class Girls Available
 

Hair and nail disorders guide

  • 1. Disorder of hair and nails 1) Fungal finger onycholysis usually results from: A. Fusarium spp B. Trichophyton rubrum C. Candida albicansCorrect Choice D. Scopulariopsis brevicaulis E. Trichophyton interdigitale It is widely accepted that C. albicans acts as colonizer that has found an ideal environment. 2) The visible portion of the nail matrix is called the: A. Hyponychium B. Eponychium C. LunulaCorrect Choice D. Nail plate E. Cuticle The nail unit is comprised of six major components including the nail matrix, nail platic, cuticle, nail bed, anchoring portion, and the framing portion (proximal/lateral/distal folds). The lunula is a component of the distal matrix. It is grayish white because of the nature of the keratinization of its epithelium 3) Steatocystoma multiplex and natal teeth are associated with which of the following conditions? A. Rubenstein-Taybi syndrome B. Nail-patella syndrome C. Jackson-Sertole pachyonychia congenita (Type 2) Correct Choice D. Jadassohn-Lewandowsky pachyonychia congenita (Type 1) E. Coffin-Siris syndrome Jackson-Sertole pachyonychia congenita (Type 2) is characterized by thickening of the nailbed and plate, steatocystoma multiplex, and natal teeth 4) Atrophy of the isthmus is a characteristic histologic finding of which of the following conditions? A. Folliculitis Decalvans B. Parry-Romberg syndrome C. Discoid lupus erythematosus D. Lichen planopilaris Correct Choice E. Acne Keloidalis Nuchae Lichen planopilaris is characterized by perifollicular lymphocytic infiltration with atrophy of the isthmus
  • 2. 5) A patient with diffuse severe sudden hair loss developing 3 months after hospitalization for septicemia likely has which of the following conditions? A. Uncombable hair syndrome B. Catagen effluvium C. Loose anagen syndrome D. Anagen effluvium E. Telogen effluvium Correct Choice Telogen effluvium typically ensues stressful events after a 2-3 month period 6) Primary total dystrophic onychomycosis is due to: A. Trichophyton rubrum B. Candida sppCorrect Choice C. Fusarium D. Scytalidium dimidiatum E. Scopulariopsis brevicaulis The main characteristic of this immunologic disorder the simultaneous involvement of all the tissues of the nail unit 7) An 18 year-old girl who was hospitalized last month after a serious car accident is noted to have white transverse grooves on her fingernails and toenails. The most likely diagnosis is: A. Half-and-half nails B. Twenty-nail dystrophy C. Terry’s nails D. Mees’ lines E. Beau’s lines Correct Choice Beau’s lines are transverse grooves in the nails. They are caused by a generalized systemic condition which disrupts nail formation. Conditions may include infection, myocardial infection, neurologic events and cytotoxic medications. 8) Papular atrichia is caused by mutations in which gene? A. Patched B. Wingless C. Fox forehead D. Hairless Correct Choice E. Distal-less homebobox Hairless gene mutations cause papular atrichia. 9) This nail finding is characteristically associated with what disorder?
  • 3. A. Congenital heart failure B. Hyperthyroidism C. Cirrhosis D. Chronic renal failureCorrect Choice E. Anemia Lindsay’s nail is characteristically associated with chronic renal failure 10) Koilonychia is most commonly seen in association with: A. Progressive systemic sclerosis B. Renal disease C. Impaired iron metabolismCorrect Choice D. Pulmonary disease E. Alopecia areata In koilonychia (spoon nails), the nail is a concave shape with raised lateral edges. Iron-deficiency anemia and Plummer-Vinson syndrome have been associated with koilonychia. Involvement of only the first three fingernails is suggestive of an occupational cause. In early infancy, koilonychia is a normal finding. Koilonychia is also seen in hemochromatosis, an autosomal recessive condition associated with increased intestinal iron absorption and increased iron deposition leading to metallic-grey hyperpigmentation of the skin, hepatomegaly, diabetes, hypogonadism, polyarthritis, and cardiac abnormalities 11) What condition most likely describes a hair defect with ruffled cuticle in 4-year old female with blond hair? A. Loose anagen hair syndromeCorrect Choice B. Pili annulati C. Pili torti D. Wooly hair E. Monilethrix Loose anagen hair syndrom occurs primarily in young, females with blond hair. The defect results in early keratinization of the inner root sheath which leads to poor anchoring. Examination of hair shaft reveals ruffled proximal cuticle 12) How long should last the wash-out period before performing mycologic investigations in a clinically recurrent onychomycosis treated by terbinafine, itraconazole or fluconazole: A. Four months B. Two months C. Three monthsCorrect Choice D. Six months E. One month This is an interesting point, because we know that these systemic antifungals are retained in the keratin of the toenails up to six months after the treatment has been stopped. Therefore this answer could have been debatable were it not for the presence of fungi in the nail
  • 4. bed (which is the primary location of the disease) and where the scraping should be properly performed for mycologic testing after removal of a piece of nail as proximal as possible 13) Which of the following is NOT a cause of red lunulae? A. EndocarditisCorrect Choice B. Carbon monoxide poisoning C. Rheumatoid arthritis D. Imuran E. Psoriasis Endocarditis is not a cause of red lunulae. Alopecia areata, psoriasis, prednisone for connective tissue diseases, carbon monoxide poisoning, cardiac failure, SLE, RA, COPD, CO2, cirrhosis, imuran and radiation are the causes of this condition 14) The "ragged cuticle" seen in dermatomyositis is also known as: A. Emperipolesis B. Plummers sign C. ventral pterygium D. Candy-cane nails E. Samitz signCorrect Choice Cuticular fraying from dermatomyositis is called the Samitz sign or "ragged cuticle". The other options are not associated with dermatomyositis. A ventral pterygium can be seen in systemic sclerosis. There is no such sign as Plummers sign. Candy-cane nails are seen in Darier's disease. Emperipolesis is seen in Rosai-Dorfman disease and is not a nail finding 15) Acute paronychia is most commonly caused by which of the following organisms? A. Trichophyton rubrum B. Staphylococcus aureus Correct Choice C. Candida tropicalis D. Candida albicans E. Pseudomonas aeruginosa Staphylococcus aureus is the most common cause of acute paronychia 16) The combination of poliosis, uveitis, deafness, and vitiligo are characteristic of which of the following conditions? A. Vogt-Koyanagi-Harada syndrome Correct Choice B. Cornelia de Lange Syndrome C. Piebaldism D. Waardernburg syndrome E. Cronkhite-Canada syndrome Vogt-Koyanagi-Harada syndrome is characterized by poliosis, uveitis, deafness, and Vitiligo
  • 5. 17) A patient with curly hair that straightened after puberty, enamel hypoplasia, dental pits, and increased bone density likely has a mutation in which of the following genes? A. SPINK 5 B. Distal-less homeobox-3 gene (DLX3) Correct Choice C. SLURP 1 D. Bone morphogenetic protein type 2 E. Vascular-endothelial growth factor receptor 3 Distal-less homeobox-3 gene (DLX3) mutations cause trichodentoosseous syndrome, which is characterized by curly hair that straightens in the 2nd-3rd decades, enamel hypoplasia, dental pits, and increased bone density 18) Argininosuccinic aciduria is characteristically associated with this hair finding –name the condition pictured below: A. Trichoschisis B. Monilethrix C. Pili trianguli et canaliculi D. Trichorrhexis nodosa Correct Choice E. Trichorrhexis invaginata Trichorrhexis nodosa is a finding in argininosuccinic aciduria 19) Signs of virilization include all of the following EXCEPT: A. Increased muscle mass in the limb girdles B. Clitoromegaly C. Temporal balding D. Centripetal obesityCorrect Choice E. Acne Centripetal obesity is a sign of cortisol excess not of virilization. In addition to the those listed above, irregular or absent menses and deepening of the voice are signs of virilization 20) Distal subungual onychomycosis is most often caused by: A. Epidermophyton floccosum B. Trichophyton megninii C. Trichophyton mentagrophytes D. Trichophyton rubrumCorrect Choice E. Trichophyton schoenleinii Trichophyton rubrum is the most common organism involved in distal subungual onychomycosis. 21) Yellow nails can be seen in each of the following syndromes except:
  • 6. A. Bronchiectasis B. HyperlipidemiaCorrect Choice C. Thyroid disease D. Penicillamine E. Tobacco use Yellow discoloration of the nail plate can be associated with bronchiectasis or underlying pulmonary disease such as COPD. Other associated conditions include lymphedema, rheumatoid arthritis, thyroid disease, malignancies, penicillamine, and chronic sinusitis or bronchitis. Hyperlipidemia is not associated with nail changes 22) Lindsay's nails (distal nail normal, proximal nail white) is characteristic of: A. Plummer-Vinson syndrome B. LEOPARD syndrome C. Hemochromatosis D. Chronic renal failureCorrect Choice E. Ectodermal dysplasia Lindsay's nails are characteristic of chronic renal failure and indicate nailbed edema. The remaining options are causes of koilonychia, a spooning defect of the nail 23) A woman with hypopigmented in lines of Blaschko and scarring alopecia likely suffers which of the following conditions? A. Chondrodysplasis punctata B. Anhidrotic Ectodermal Dysplasia C. Bloch-Sulzberger Syndrome Correct Choice D. Rothmund-Thomson Syndrome E. Focal Dermal Hypoplasia Bloch-Sulzberger Syndrome (incontinentia pigmenti) 24) The most common cause for superficial type onychomycosis is: A. Epidermophyton floccosum B. Scopulariopsis brevicalis C. Trichophyton rubrum D. Fusarium oxysporum E. Trichophyton mentagrophytesCorrect Choice T. mentagrophytes is the most common cause of superficial type onychomycosis. Other causes include: Fusarium, Acremonium and Aspergillus spp 25) Congenital generalized hypertrichosis is associated with which of the following symptoms? A. Methimazole teratogenicity
  • 7. B. Gingival fibromatosis Correct Choice C. Androgen-secreting ovarian tumors D. Pituitary insufficiency E. Distichiasis Gingival fibromatosis. Congenital Generalized Hypertrichosis with Gingival Fibromatosis is a genetic condition with hypertrichosis on the face and upper body and gingival hyperplasia and fibromatosis 26) A significantly increased number of "club hairs" on a pull test is indicative of: A. Anagen effluvium B. Alopecia areata C. Angrogenetic alopecia D. Telogen effluviumCorrect Choice E. Normal hair anatomy Telogen hairs are characteristically club shaped. An increased number of telogen hairs on hair pull would be suggestive of telogen effluvium. Anagen hairs have a curled appearance at the root. While a small number of telogen hairs are normal, a significantly increased number would not be normal hair anatomy 27) Trichobezoar is is a potentially life-threatening complication of: A. Discoid lupus B. Anhidrotic ectodermal dysplasia C. Alopecia areata D. Lichen planopilaris E. Trichotillomania Correct Choice NEEDS EXPLANATIONS 28) Which of the following is an example of an apparent leukonychia? A. Mee's lines B. Terry's lines C. Onychoschizia D. Hutchinson's lines E. Muehrcke's linesCorrect Choice An apparent leukonychia disappears with pressure and is a problem with nail bed, not nail plate. Apparent leukonychia may be indicative of overall health compromise. Muehrcke's lines are paired white bands and are a type of apparent leukonychia 29) Endonyx onychomycosis is due to: A. Trichophyton soudanenseCorrect Choice B. Trichophyton rubrum
  • 8. C. Scytalidium dimidiatum D. Scopuliaropsis E. Trichophyton mentagrophytes Endonyx onychomycosis may also be observed with T. violaceum. This is the equivalent of endothrix infection of the hair associated with tinea capitis 30) A patient with yellow nail syndrome should be evaluated for systemic disease involving which organ system? A. Hematologic B. Hepatic C. Pulmonary Correct Choice D. Cardiovascular E. Renal Yellow nail syndrome is characterized by slowly growing, yellow nails that are excessively curved. The lunulae and cuticles are absent. It has been associated with a array of pulmonary diseases including tuberculosis, asthma, pleural effusion, bronchiectasis, sinusitis, bronchitis, and COPD 31) The defect associated with the hair finding shown in the image is: A. hHB6/hHB1Correct Choice B. hHB8/hHB3 C. hHB10/hHB5 D. hHB9/hHB4 E. hHB7/hHB2 hHB6/hHB1 keratins are associated with Monilethrix. Other findings include: short, brittle hair, extensor keratosis pilaris, mental retardation, syndactyly, cataracts, and nail/teeth abnormalities 32) Which of the following syndromes does not have Dorsal pterygium as a feature: A. Lesch-Nyhan syndrome B. Lamellar ichthyosisCorrect Choice C. Cicatricial pemphigoid D. Chronic GVHD E. Lichen planus Dorsal pterygium (scarring of the proximal nail fold) is caused by lichen planus, acrosclerosis, onychotillomania, Lesch-Nyhan syndrome, chronic GVHD, SJS/TEN and cicatricial pemphigoid. All of the listed items are correct except lamellar ichthyosis. It does not have this nail finding. 33) Which part of the matrix gives rise to the dorsal nail plate? A. Distal matrix B. Dorsal matrix C. Ventral matrix
  • 9. D. Lateral matrix E. Proximal matrix Correct Choice The proximal matrix gives rise to the dorsal nail plate, while the distal matrix gives rise to the ventral nail plate 34) Yellow nail syndrome can be associated with: A. D-penicillamine use in patients with rheumatoid arthritisCorrect Choice B. Alopecia areata C. Wilson’s disease D. Doxorubicin E. Minocycline use in acne patients Answer: A. D-penicillamine use in patients with rheumatoid arthritis. Yellow nail syndrome has been described in patients with rheumatoid arthritis receiving thiol compounds including D-penicillamine and bucillamine. Reversal of the condition has been described after discontinuation of the drug 35) A patient with cirrhosis has noticed that the proximal aspect of his nails have become white. This characteristic finding is called: A. Muehrcke's lines B. Mees' lines C. Terry's linesCorrect Choice D. Beau's lines E. Half-and-half nails Terry’s lines are described as an abnormal white appearance of the nail except for the most distal portion. It has been described in association with cirrhosis, congestive heart failure, and diabetes mellitus 36) A defect in which part of the nail cause a true leukonychia? A. Lunula B. Nail plateCorrect Choice C. Hyponychium D. Nail bed E. Cuticle True leukonychia is caused by an acute stress to the nail and manifests in the nail plate. Examples include Terry's nails and Mee's lines. Apparent leukonychia, on the other hand, are a result of problems in the nail bed and are indicative of overall health compromise. Examples of apparent leukonychia include Lindsay's and Muehrcke's nail 37) Tay’s syndrome is characterized by this hair condition pictured below – identify the condition: A. Monilethrix B. TrichoschisisCorrect Choice
  • 10. C. Trichorrhexis nodosa D. Trichorrhexis invaginata E. Pili trianguli et canaliculi Tay’s syndrome is also known as trichothiodystrophy which is characterized by trichoschisis or clean fracture of the hair shaft 38) A patient with wooly hair, diffuse palmoplantar keratoderma, and right ventricular arrhythmogenic cardiomyopathy likely has a mutation in which of the following genes? A. HHB6 B. Plakophilin C. SPINK 5 D. Plakoglobin Correct Choice E. SLURP 1 Naxos syndrome is caused by mutations in the plakoglobin gene. Its clinical features include wooly hair, diffuse palmoplantar keratoderma, and right ventricular arrhythmogenic cardiomyopathy 39) Black nail is caused by: A. Proteus mirabilisCorrect Choice B. Staphylococcus aureus C. Hortaea werneckii D. Pseudomonas spp. E. Trichophyton mentagrophytes Nail infections with Proteus mirabilis. T. mentag causes white discoloration of the nail and Pseudomonas will cause a green or yellow/green discoloration. Hortaea werneckii causes tinea nigra. S. aureus does not cause black nails 40) The combination of a low-set hairline and synophrys is seen in which of the following conditions? A. Noonan Syndrome B. Turner Syndrome C. Waardenburg's syndrome D. Cornelia de Lange Syndrome Correct Choice E. Werner syndrome Explanation 41) Pili trianguli et canaliculi is the characteristic hair fiding in which of the following disoders? A. Monilethrix B. Naxos syndrome C. Anhidrotic ectodermal dysplasia
  • 11. D. Pili annulati E. Uncombable hair syndrome Correct Choice Pili trianguli et canaliculi, characterized by a triangular hair shaft with a central longitudinal groove, is the finding seen in uncombable hair syndrome 42) The most common cultured agent in chronic paronychia associated with a black hue of the lateral edge of the nail plate is: A. Trichophyton rubrum B. gram-negative bacteria C. Scytalidium dimidiatum D. Scopulariopsis brevicaulis E. Candida spp.Correct Choice Chronic paronychia is likely a condition that represents a contact reaction to allergens or irritants. It often is superinfected with candida. This underlies the importance of treatment with both an antifungal and a topical steroid 43) Cutaneous signs of cortisol excess include all of the following EXCEPT: A. Atrophic/fragile skin B. Dorsocervical/Supraclavicular fat pads C. Striae distensae D. Plethora E. Peripheral obesityCorrect Choice All of the listed options are signs of cortisol excess except peripheral obesity. The most common pattern of obesity in patients with cortisol excess is centripetal or central obesity 44) Which genetic defect could be expected in an individual with these nail findings in addition to short stature, capillary malformations, beaked nose, mental retardation, and cryptorchidism? A. ectodysplasin B. hHB1/hHB6 C. GNAS D. CREBBPCorrect Choice E. ERCC6 Rubenstein-Taybi syndrome is a genetic disorder caused by mutations in the gene encoding CREB binding protein. This protein acts as a nuclear protein involved in coactivating cAMP regulated gene expression. Clinically, patients have capillary capillary malformations, beaked nose, mental retardation, cryptorchidism, and broad thumbs 45) Which autoantibody is associated with these cuticular changes, shawl sign, and overall favorable prognosis? A. Scl-70 B. High titered ANA with speckled pattern
  • 12. C. Mi-2Correct Choice D. Ro E. Jo-1 Dermatomyositis has many different cutaneous manifestations including heliotrope rash, Gottron's papules and sign, mechanic's hand and poikiloderma atrophicans vasculare. Antibodies to Mi-2 in dermatomyositis are associated with cuticular changes, shawl sign and a overall favorable prognosis 46) The most common nail infection that causes white superficial/proximal white subungal onychomycosis is: A. Trichophyton rubrumCorrect Choice B. Trichophyton mengatrophytes C. Scatylidium hyalinum D. Aspergillus spp E. Hendersonula toruloidea T. rubrum is the most common cause of white superficial or proximal white subungal onychomycosis. This can also be caused by aspergillus. T. mentag causes superficial onychomycosis and less commonly distal subungal type. Hendersonula and Scatylidium cause lateral invasion with paronychia 47) Acquired progressive kinking of hair is a complication of which of the following medications? A. Slfonamides B. Antimetabolites C. Antimalarials D. Dapsone E. Retinoids Correct Choice Retinoids can cause acquired progressive kinking of hair 48) Follicular atrophoderma is a feature of which of the following conditions? A. Papular atrichia B. Tricho-rhino-phalangeal syndrome C. Menkes kinky hair syndrome D. Tricho-dento-osseous syndrome E. Bazex syndrome Correct Choice Bazex syndrome is characterized by follicular atrophoderma, hypotrichosis, hypohidrosis, and mutiple BCCs of face. 49) Proximal white subungual onychomycosis with polydactylous involvement needs at least one laboratory test: A. HIVCorrect Choice B. T-helper CD4 lymphocytes
  • 13. C. Blood count D. Sedimentation rate E. Liver enzymes This type of onychomycosis has a prognostic value: It means that the CD4 lymphocytes are below 450. 50) Yellow nail syndrome is associated with which of the following? A. Diabetes mellitus B. Dermatophyte infection C. Multiple myeloma D. Lymphedema and bronchiectasis Correct Choice E. Panhypopituitarism Lymphedema and bronchiectasis are associated with yellow nail syndrome 51) This hair disorder and mutations in ATP7A (MNK) gene are found in what genodermatosis? A. Crandall's syndrome B. Bjornstad syndrome C. Trichothiodystrophy D. Menke's kinky hair syndrome Correct Choice E. Bazex's follicular atrophoderma Menke's kinky hair syndrome is characterized by pili torti and mutations in APT7A (MNK) gene. Bjornstad syndrome, Crandall’s syndrome and Bazex’s follicular atrophoderma are associated with pili torti but are not associated with this mutation 52) Which of the following conditions with hair abnormalities is caused by mutations in a DNA helicases? A. Anhidrotic Ectodermal Dysplasia B. KID Syndrome C. Rothmund-Thomson Syndrome Correct Choice D. Hidrotic Ectodermal Dysplasia E. Incontinentia Pigmenti Explanation 53) Pathognomonic nails changes in HOOD syndrome or Hereditary osteoonychodysplasia are A. Pterygium inversum B. Triangular lunulaeCorrect Choice C. Yellow nails D. Red and white longitudinal banding
  • 14. E. Pincer nails In the HOOD syndrome (Hereditary osteoonychodysplasia) or the Nail-Patella syndrome, patients have different degrees of nail dysplasia. In general, the thumbs are most severely involved and nail disease decreases in severity from the second to the fifth digits. Characteristic nail changes include longitudinal ridging, koilonychia, and splitting. Toenails are rarely involved. Pathognomonic nail change is the V shaped triangular lunulae with the distal peak in the midline 54) What is the mode of inheritance of this disorder of the hair shaft? A. Sporadic B. Autosomal recessive C. X-linked recessive D. Autosomal dominantCorrect Choice E. X-linked dominant Monilethrix is an autosomal dominant condition with variable penetrance. The hair shaft has an elliptical or beaded hair appearance 55) Triangular lunulae are: A. associated with a renal cell carcinoma B. associated with hyperplastic patellae C. part of the syndrome caused by a LMX1b mutationCorrect Choice D. associated with a mutation of SERCA2 E. associated with a mutation of ATP2C1 Triangular lunulae are part of the Nail-Patella syndrome. Findings in this syndrome include LMX1b mutation, hypoplastic or absent patellae, nephropathy, a "Lester Iris" - hyperpigmentation of the pupillary margin, cataracts, heterochromia irides and glaucoma. ATP2C1 and SERCA2 are associated with Hailey-Hailey and Darier-White disease respectively. 56) Fraying of the cuticles is a sign of which of the following conditions? A. Lead poisoning B. Lupus erythematosus C. Dermatomyositis Correct Choice D. Systemic sclerosis E. Hypothyroidism Dermatomyositis can be associated with fraying of the cuticles (Samitz sign). 57) Frontal bossing, saddle nose, hypoplastic midface, peg shaped or conical teeth and hypopigmented short sparse scalp and body hair are prominent features of which of the following A. Hidrotic ectodermal dysplasia B. Arginosuccinic aciduria C. Monilethrix
  • 15. D. Christ Siemens Touraine syndromeCorrect Choice E. Pachyonychia congenita Chrsit Siemens Touraine syndrome also known as Hypohidrotic ectodermal dysplaisa or anhidrotic ectodermal dysplasia is X-linked recessive disroder that presents in infancy to early childhood. Features include hypo-anhidrosis with increased body temperature, hypopigmented sparse scalp and body hair, frontal bossing, saddle nose, hypoplastic midface, abnormal ears, hypo-anodontia, and increased bronchopulmonary inections. Cloustons syndrome or hidrotic ectodermal dysplasia is AD and associated with palmoplantar keratoderma, nail dystrophy, short sparse scalp hair, and tufting of the terminal phalanges. Arginosuccinic aciduria is associated with seizures, trichorrhexis nodosa, failure to thrive, and hyperammonemia. Monelothrix is associated with beaded hairs and keratosis pilaris. Pachyonychia congenita is associated with nail dystrophy and focal palmoplantar keratoderma 58) Which of the following syndromes does NOT have the finding shown in the image as part of the spectrum? A. CrandallCorrect Choice B. Argininosuccinic aciduria C. Citrullinemia D. Menke's Kinky Hair E. Netherton Crandall syndrome has pili torti as its hair finding. The other options all have trichorrhexis nodosa as one of the hair findings in the syndrome. Trichorrhexis nodosa is the fraying of hair ends so that the resemble a "broom-stick" or "paint brush" 59) Which of the following is caused by mutations in gap junction proteins? A. Hidrotic ectodermal dysplasiaCorrect Choice B. Anhidrotic ectodermal dysplasia C. Netherton syndrome D. Naxos syndrome E. Rothmund-Thompson syndrome Hidrotic ectodermal dysplasia is caused by mutations in connexin 30, which is a gap junction protein 60) Proximal white subungual onychomycosis associated with paronychia is never due to: A. Fusarium spp B. Candida spp.Correct Choice C. Scopulariopsis brevicaulis spp D. Acremonium spp E. Aspergillus spp. All of the molds indicated, here, may be responsible for this condition, but principally 2,3,4 61) Muehrcke's nails are most often associated with A. Hyperalbuminemia
  • 16. B. Hyperthyroidism C. HypoalbuminemiaCorrect Choice D. Hypothyroidism E. None of these answers are correct Muehrcke's nails are most associated with hypoalbuminemia. They represent transverse double white lines that are the abnormality of the vascular bed, probably a localized edematous state secondary to the hypoalbuminemia 62) Anonychia is not a feature of which of the following syndromes? A. Hidrotic ectodermal dysplasiaCorrect Choice B. DOOR syndrome C. Nail-Patella syndrome D. Coffin-Siris syndrome E. COIF syndrome Hidrotic ectodermal dysplasia features hyperconvex nails, micronychia and nail dystrophy. All of the other listed options can cause anonychia. COIF syndrome is Conginital Onychodysplasia of the Index Finger and DOOR syndrome is Deafness, Congenital Onychodystrophy Recessive Form. Coffin-Siris syndrome is characterized by: nail onychodysplasia or aplasia (usually of the fifth finger or toe), coarse facies including bushy eyebrows, scant scalp hair, full lips, and microcephaly, mental/growth deficiency, short distal phalanges, and other abnormalities 63) A 4 year-old boy has a single circular alopecic atrophic patch on the vertex of the scalp. This most likely is due to which of the following conditions? A. Triangular Alopecia B. Aplasia cutis congenita Correct Choice C. Congenital syphilis D. Keratosis Follicularis Spinulosa Decalvans E. Morphea en coup de sabre Aplasia cutis congenita often affects the vertex of the scalp and heals with a circular atrophic alopecic scar 64) Splinter hemorrhage of the nail can be seen with which of the following parasitic infections? A. Gnathostomiasis B. Dracunculiasis C. Trichinosis Correct Choice D. Scabies E. Sparganosis Trichinosis can cause splinter hemorrhage of the nails. 65) What is the gene defect that causes red hair?
  • 17. A. Melanocortin 1 receptorCorrect Choice B. P protein C. Tyrosine aminotransferase D. Tyrosinase E. Tyrosinase-related protein 1 Defects in melanocortin 1 receptor lead to the phenotypic appearance of red hair. Individuals with red hair have increased ratio of pheomelanin to eumelanin and are at increased risk for melanoma 66) Chronic paronychia is most commonly caused by which of the following organisms? A. Staphylococcus aureus B. Pseudomonas aeruginosa C. Candida albicans Correct Choice D. Candida tropicalis E. Trichophyton rubrum Candida albicans is the most common pathogen associated with chronic paronychia 67) The common culprit of proximal white subungual onychomycosis is : A. Fusarium spp B. Candida albicans C. Scydalidium dimidiatum D. Trichophyton rubrumCorrect Choice E. Trichophyton interdigitale T. rubrum is responsible for more than 95% of these cases. 68) The syndrome including mental and physical retardation, convulsions, episodic unconsciousness, liver enlargement, skin lesions, and dry and brittle hair showing trichorrhexis nodosa microscopically and fluorescing red. is: A. Citrullinemia B. Argininosuccinic aciduriaCorrect Choice C. Crandall's syndrome D. Bjornstad's syndrome E. Bazex Follicular Atrophoderma Argininosuccinic aciduria has the features listed including trichorrhexis nodosa and red fluorescence of the hair. Bazex's follicular atrophoderma and Crandall syndrome both have findings of pili torti and citrullinemia has trichorrhexis nodosa. None of the other options have red fluorescence as a feature 69) The epithelium that lies on the volar surface of the digit is the: A. Hyponychium Correct Choice
  • 18. B. Eponychium C. Nail plate D. Nail bed E. Nail matrix The epithelium that lies on the volar surface of the digit is the hyponychium. The nail bed is the actual nail made up of tightly packed onychocytes. The nail matrix is the epithelium which starts mid distal phalanx which generated the nail plate. It keratinizes without a granular layer and determines the thickness of the nail plate. The nail bed is the thin epithelium immeduately beneath the nail plate 70) Triangular lunulae are seen in what disorder? A. Gorlin’s syndrome B. Dyskeratosis congentia C. Nail-patella syndrome Correct Choice D. Neurofibromatosis II E. Papillon-Lefevre syndrome Triangular lunulae are seen in nail-patella syndrome which is characterized by mutation in LMX1b 71) Which of the following are characteristics of Trichorhinophalangeal syndrome? A. Brittle hair, short stature, cerebellar ataxia B. Shortened phalanges, sparse hair, bulbous noseCorrect Choice C. Kinky hair, frontal bossing, small widely spaced teeth with poor enamel D. Sparse fine hair, Short extremities, Immunodeficiency E. Palmoplantar keratoderma, trichorrhexis nodosa, sinus infections Characteristic features of trichorhinophalangeal syndrome are shortened phalanges, sparse hair, and bulbous nose 72) Blue lunulae are associated with each of the following except: A. Wilson’s disease B. Phenolphthalein C. Argyria D. Quinacrine E. Cardiac failureCorrect Choice Blue lunulae have been reported in association with argyria, Wilson’s disease, hereditary acrolabial telangectasia, paronychia, phenolphthalein, quinacrine, topical bichloride, and mercury exposure 73) The best test for Cushing syndrome is: A. Plasma LH B. Plasma DHEA C. Plasma prolactin D. Overnight dexamethasone suppresion testCorrect Choice
  • 19. E. Plasma testosterone The overnight dexamethasone suppression test is the test of choice for diagnosis of Cushing syndrome. In this test, a dose of dexamethasone is given at 11pm and the plasma cortisol level is drawn the following morning. If the Cushing syndrome is from an adrenal tumor or an ectopic ACTH producing tumor, there is no change in cortisol secretion. A normal response is suppression of the cortisol level by the extra dexamethasone. If the Cushings is from a pituitary tumor, at low doses of dexamethasone there are no changes in cortisol levels while at high doses of dexamethasone there will be a normal suppression. The other listed items are not used for diagnosis of Cushing syndrome. 74) Which part of the nail complex results in nail pits? A. Proximal nail fold B. Nail bed C. Proximal nail matrixCorrect Choice D. Nail plate E. Dorsal nail matrix Pitting of the nails are punctate depressions that migrate distally and result from defect in the proximal nail matrix. It may be seen in conditions such as psoriasis, eczema and alopecia areata 75) Spotted red lunulae are absent in which of the following conditions? A. Rheumatoid arthritis B. Systemic lupus erythematosus C. Keratosis follicularisCorrect Choice D. Lichen planus E. Alopecia areata Keratosis follicularis has nail findings of v-shaped nicking of the distal nail, subungual hyperkeratosis and red/white alternating longitudinal bands. In addition to those listed above, psoriasis can also cause spotted red lunulae 76) A patient with sparse hair, a pear-shaped broad nose and cone-shaped epiphyses likely suffers which of the following conditions? A. Hay-Wells Syndrome B. Focal Dermal Hypoplasia (Goltz syndrome) C. Trichodentoosseous Syndrome D. Tricho-rhino-phalangeal Syndrome Correct Choice E. Ectrodactyly-Ectodermal dysplasia-Clefting (EEC) Tricho-rhino-phalangeal Syndrome is characterized by sparse hair, a pear-shaped broad nose and cone-shaped epiphyses 77) A patient with diffuse hair loss developing after a thallium scan likely has which of the following conditions? A. Loose anagen syndrome
  • 20. B. Telogen effluvium C. Anagen effluvium Correct Choice D. Uncombable hair syndrome E. Catagen effluvium Anagen effluvium can result from infusions of thallium 78) Yellow nail syndrome has been associated with use of which of the following treatments for Rheumatoid Arthritis? A. Infliximab B. D-penicillamineCorrect Choice C. Adalimumab D. Methotrexate E. Etanercept D-penicillamine has been associated with the development of yellow nail syndrome in patiens with rheumatoid arthritis. The other options are used to treat RA, but have not been associated with development of this syndrome 79) Blue lunulae are characteristic of which disease? A. Clubbing B. Rubenstein-Taybi syndrome C. Yellow nail syndrome D. Wilson's disease Correct Choice E. Hypertrophic pulmonary osteoarthropathy Wilson's disease is associated with characteristic blue lunulae 80) Which genodermatosis characterized by broad thumbs and this nail disorder pictured below? A. Rubenstein-Taybi syndrome Correct Choice B. Proteus syndrome C. Tuberous sclerosis D. Rothmund-Thompson E. Pachyonychia congenita Rubensteom-Taybi syndrome is characterized by broad thumbs and broad nails or brachyonychia. This syndrome is characterized by mutations in CREB-binding protein 81) What is the common cause of the “one hand-two-foot” syndrome? A. Trichophyton rubrumCorrect Choice B. Trichophyton mentagrophytes ( var. Interdigitale) C. Candida albicans
  • 21. D. Scytalidium dimidiatum E. Scopulariopsis brevicaulis This condition is due to T. rubrum in 90% of the cases , T. interdigitale and Scytalidium dimidiatum share the 10% left 82) Regarding androgens in women, which of the following statements is NOT correct? A. Eyebrows, eyelashes and vellus hairs are androgen-dependentCorrect Choice B. There are no differences in eyelashes, eyebrows and vellus hair-bearing areas in men and women C. Testosterone binds the androgen receptor D. The hair follicle requires conversion of testosterone to dihydrotestosterone for expression of androgen action E. Dihydrotestosterone binds the androgen receptor Only testosterone and dihydrotestosterone bind the androgen receptor, Eyebrows, eyelashes and vellus hairs are NOT androgen-dependent, thus there are no differences between these areas in men and women, and the hair follicle requires conversion of testosterone to dihydrotestosterone for expression of androgen action 83) Ventral pterygium pictured below is characteristically associated with what disorder? A. Chronic GVHD B. Cicatricial pemphigoid C. SJS/TEN D. Lichen planus E. Systemic sclerosisCorrect Choice Ventral pterygium is most characteristically associated with systemic sclerosis. All other choices are associated with dorsal pterygium 84) Arsenical poisoning is associated with what nail finding? A. Oil spots B. Onycholysis C. Muehrcke's lines D. Mee's lines Correct Choice E. Hapalonychia Arsenical poisoning is characteristically associated with Mee's lines 85) The hair finding characterized by an invagination of the distal hair shaft into the cup formed by the proximal hair shaft is: A. Pili torti B. Pili triangulati C. Trichoschisis D. Trichorrhexis invaginataCorrect Choice
  • 22. E. Trichorrhexis nodosa The above description is that of trichorrhexis invaginata, which is characteristic of Netherton syndrome. Other findings include: ichthyosis linearis circumflexa and atopy. It is caused by the mutation of the serine protease inhibitor, SPINK5 86) Psoriatic onycholysis is caused by psoriasis in which of the following nail subunits? A. Proximal nail fold B. Hyponychium C. Nail plate D. Nail bed Correct Choice E. Nail matrix Nail bed involvement by psoriasis often causes onycholysis 87) A patient with synophrys, dystopia canthorum, and heterochromia irides likely has which of the following hair abnormalities? A. Trichorrhexis invaginata B. Scarring alopecia C. White forelock Correct Choice D. Diffuse thinning E. Trichorrhexis nodosa White forelock is a common feature of Waardenburg syndrome 88) Mee's lines are: A. double white transverse lines from abnormal vascular bed B. local or diffuse hyperkeratotic tissue that develops on the lateral or proximal nail folds C. transverse white lines that affect all nails, grow out with nail growthCorrect Choice D. vertical black lines on a single or multiple nails E. brownish macules beneath the nail plate Mee's lines are transverse white lines that affect all nails, grow out with nail growth. They are seen in arsenic poisoning, rheumatic fever, congenital heart failure, leprosy and with significant systemic disease. Brownish macules beneath the nail plate are oil spots, seen in psoriasis. Onychophosis is the local or diffuse hyperkeratotic tissue that develops on the lateral or proximal nail folds. The double white transverse lines from abnormal vascular bed are Muehrcke's lines which are caused by a nephrotic syndrome, low albumin, liver disease or malnutrition 89) Highest graft survival in hair transplantation is achieved through the use of: A. Micrografts B. Follicular unitsCorrect Choice C. Single hairs D. Minigrafts
  • 23. E. 4-5 mm plugs Seager et. al examined the survival rate of single hair grafts and compared them to follicular unit grafts in a single patient. They found that there was a much higher survival rate in the follicular unit grafts. They hypothesized that the extra tissue surrounding the follicular unit grafts protected them from crush injury 90) This characteristic nail finding is seen in what disorder? A. Darier's diseaseCorrect Choice B. Coffin-Siris syndrome C. Lichen planus D. COIF syndrome E. Pachyonychia congenita Alternating red and white longitudinal bands on the nail are characteristic of Darier’s disease. Other nail findings include subungual hyperkeratosis and distal v-shaped nicking 91) A commonly known cause of splinter hemorrhages in the nail is endocarditis. Which of the following would be unlikely to cause splinter hemorrhages? A. Trichinosis B. Vasculitis C. Rheumatoid arthritisCorrect Choice D. Trauma E. Psoriasis Rheumatoid arthritis is not a cause of splinter hemorrhages of the nail. Endocarditis is the most commonly thought of cause of splinter hemorrhages. Trichinosis, trauma, psoriasis and vasculitis also can be causes of this also and should be considered when splinter hemorrhages are seen 92) The combination of ankyloblepharon, ectodermal dysplasia, and cleft palate with wiry sparse hair is characteristic of which of the following syndromes? A. Tricho-rhino-phalangeal Syndrome B. Focal Dermal Hypoplasia (Goltz syndrome) C. Trichodentoosseous Syndrome D. Ectrodactyly-Ectodermal dysplasia-Clefting (EEC) E. Hay-Wells Syndrome Correct Choice Hay-Wells Syndrome, also known as AEC (Ankyloblepharon-Ectodermal dysplasia-Clefting) is characterized by wiry, sparse hair or alopecia, ankyloblepharon, PPK, partial anhidrosis, cleft lip, palate, absent, and dystrophic nails 93) Distal subungual onycholysis associated with paronychia is due to: A. Scytalidium dimidiatumCorrect Choice B. Fusarium spp C. Trichophyton mentagrophytes
  • 24. D. Candida spp E. Trichophyton rubrum This mold is the only one responsible for paronychia in DLSO and is more often observed in finger than in toenails 94) A subungual hematoma covering 50% of the nailbed should be managed by which of the following? A. Trephination of the nail plate B. Needle aspiration of the hematoma C. No treatment D. Pressure dressing E. Removal of the nail plate Correct Choice Removal of the nail plate is necessary for management of hematomas covering more than 25% of the nailbed 95) Terry's nails are seen in which of the following conditions? A. Pulmonary fibrosis B. Bladder carcinoma C. Renal failure D. Aerodigestive carcinoma E. DiabetesCorrect Choice Terry's nails are described as all but the distal 2mm of each nail evenly white due to a defect in the nail bed. It can be seen in cirrhosis, congenital heart failure and diabetes 96) A patient with a history of gastrointestinal polyposis presents with alopecia, generalized pigmentation, and nail dystrophy. Which of the following is the most likely diagnosis? A. Cronkhite-Canada Syndrome Correct Choice B. Cowden syndrome C. Peutz-Jegher syndrome D. Dyskeratosis congenita E. Gardner syndrome Cronkhite-Canada Syndrome is a sporadic gastrointestinal polyposis syndrome characterized by alopecia, generalized pigmentation, and nail dystrophy 97) What is the most common hair finding in Netherton's syndrome? A. Trichorrhexis invaginataCorrect Choice B. Pili torti C. Monilethrix D. Trichoschisis
  • 25. E. Trichorrhexis nodosa Netherton syndrome is an an autosomal recessive genodermatosis that is caused by a mutation in SPINK5. The characteristic findings include trichorrhexis invaginata, ichthyosis linearis circumflexa, and food allergy. Infants may present with erythroderma 98) Transverse white bands on one or two nails is caused by: A. trauma to the matrixCorrect Choice B. psoriasis C. systemic lupus erythematosis D. alopecia areata E. lichen planus Longitudinal leukonychia or transverse white bands are caused by trauma to the nail matrix. AA, psoriasis, SLE and LP all cause spotted red lunulae, not transverse white bands 99) Mutations in c-kit are associated with which of the following conditions? A. Piebaldism Correct Choice B. Waardenburg syndrome C. Vogt-koyanagi-harada D. Tuberous sclerosis E. Cornelia de Lange Syndrome Piebaldism is caused by mutations in c-kit 100) Which of the following medication is a cause of hirsutism without virilization? A. DiazoxideCorrect Choice B. Dinitrochlorobenzene C. Dapsone D. Diazepam E. Dantrolene Phenytoin, diazoxide, cyclosporine and hexachlorobenzene all can cause increased hair growth in patients. In addition, oral (and topical) minoxidil can cause hirsutism. The other listed medications do not cause hirsutism 101) What nutritional deficiency can lead to this nail finding? A. Vitamin B12 B. Thiamine C. IronCorrect Choice D. Magnesium E. Vitamin C Iron deficiency can lead to spoon nail deformity or koilonychias
  • 26. 102) A 42 year old woman presents with the complaint of excess hair growth on her face. She has normal menses and has recently had her "annual" exam and the note relates normal sized ovaries. What is the most logical next step? A. Refer her to endocrinology B. Order a CT of the abdomen C. Send a 21-hydroxylase enzyme deficiency test D. Check plasma levels of androstenedione and testosteroneCorrect Choice E. Biopsy from the most affected area Women with idiopathic hirsutism will have evidence of androgen excess but with normal menses, normal-sized ovaries and no evidence of tumors of adrenal or ovary and normal adrenal function. They will often have slight elevations of plasma androstenedione and testosterone. Check the blood levels of the plasma steroids would be a logical first step 103) Ferritin levels need to be, at minimum, above which of the serum levels to treat iron- deficiency related telogen effluvium? A. 30ng/dL B. 20ng/dL C. 60ng/dL D. 10ng/dL E. 40ng/dLCorrect Choice The therapeutic target for iron deficiency related telogen effluvium is greater than 40mg/dL. Other causes of telogen effluvium include thyroid dysfunction, as well as such medications as beta- blockers, antihyperlipidemic drugs, and NSAIDs 104) Plummer's nails are: A. defective keratinization on the proximal matrix B. caused by B12 deficiency C. hereditary or from shoe trauma D. caused by onycholysis from hypothyroidismCorrect Choice E. caused by cirrhosis Plummer's nails are caused by onycholysis from hypothyroidism. Pincer nails are hereditary or from show trauma, usually 1st toenails. Nail pitting is caused by defective keratinization on the proximal matrix. B12 deficiency can cause nail pigmentation 105) What determines the thickness of the nail plate? A. Nail plate B. Hyponychium C. Proximal nail fold D. Nail bed
  • 27. E. Nail matrixCorrect Choice The nail matrix determines the thickness of the nail plate 106) A patient presents with 20 nails with absent cuticles and lunulae, slow growth, dystrophic shape and a yellow hue. Which of the following findings is/are associated? A. pleural effusionsCorrect Choice B. cirrhosis C. pulmonary fibrosis D. chest pain E. upper extremity edema Yellow Nail syndrome has been associated with lower extremity edema, bronchiectasis and pleural effusions. The other listed findings are not part of Yellow Nail syndrome 107) A rapid onset of hair growth with or without accompanying virilization can occur in which of the following conditions EXCEPT: A. Ovarian cystCorrect Choice B. Adrenal carcinoma C. Arrhenoblastoma D. Krukenburg tumor of the ovary E. Adrenal adenoma All of these options except an ovarian cyst can cause a rapid onset of hair growth with or without accompanying virilization. An Arrhenoblastoma is a tumor of the ovary that secretes testosterone 108) Deposition of mucin in the hair follicle can be associated with which disease? A. Keratosis Follicularis Spinulosa Decalvans B. Adams-Oliver Syndrome C. Mycosis fungoides Correct Choice D. Perifolliculitis Capitis Abscedens et Suffodiens E. Acne Keloidalis Nuchae Mycosis fungoides can be associated with follicular mucinosis 109) A 32 year-old woman presents with the complaint shown in the image. This finding began one year ago and has slowly worsened. Her female relatives do not have similar hair patterns. Serum DHEA returns at 8544 ng/ml and serum testosterone at 3 ng/ml. What is the most likely diagnosis for cause of her hirsutism: A. Poryphria cutanea tarda B. Erythropoietic protoporphyria C. An androgen producing tumorCorrect Choice D. Stein-Leventhal syndrome E. Acute intermittent porphyria
  • 28. DHEA > 8000 ng/ml and testosterone > 2 ng/ml are suggestive of a secreting neoplasm. Porphyria cutanea tarda and Stein-Leventhal syndrome do present with hirsutism, but will not have the elevated levels of DHEA and testosterone in the serum. Acute intermittent porphyria and Erythropoietic protoporphyria do not have hypertrichosis 110) Plummer's nails are associated with which of the following disorders? A. Dermatomyositis B. Systemic sclerosis C. Lupus erythematosus D. Lead poisoning E. Hypothyroidism Correct Choice Plummer's nails are onycholysis due to hypothyroidism 111) This hair finding is caused by mutations in what gene? A. SERCA2 B. Keratin 16 C. Dyskerin D. NEMO E. SPINK 5Correct Choice Mutations in SPINK 5, a serine protease inhibitor leads to Netherton’s syndrome which is characterized by trichorrhexis invaginata or bamboo hair 112) Air spaces in the hair shaft lead to this condition pictured below – name the condition: A. Wooly hair B. Trichoschisis C. Pohl Pinkus constriction D. Pili annulatiCorrect Choice E. Pili trianguli et canaliculi Pili anulati is caused by air spaces in the hair shaft. 113) Thin hair with premature graying is characteristic of which of the following syndromes? A. Cornelia de Lange Syndrome B. Noonan Syndrome C. Werner syndrome Correct Choice D. Waardenburg's syndrome E. Turner Syndrome Explanation 114) Mutations in hair keratins hHB6 and hHB1 cause this characteristic finding pictured below – name the condition:
  • 29. A. Trichoschisis B. Trichoptilosis C. MonilethrixCorrect Choice D. Hair casts E. Tiger-tail hair Mutations in the hair keratins, hHB6 and hHB 1 cause monilethrix. 115) A patient with psoriasis has pitting of the nails. This finding is due to involvement of which part of the nail unit? A. Distal matrix B. Proximal matrixCorrect Choice C. Hyponychium D. Proximal nail fold E. Nail bed Disease in the proximal matrix is responsible for producing pitting, onychorrhexis, and Beau’s lines. Changes in the intermediate matrix can cause leukonychia, and the distal matrix may be responsible for focal onycholysis, thinned nail plate, and erythema of the lunula. 116) Björnstad syndrome is the combination of pili torti and which of the following features? A. Lentigines B. Deafness Correct Choice C. Hypogonadism D. White forelock E. Seizures Deafness and pili torti are the two cardinal features of Björnstad syndrome